Week 10 Geriatrics in Primary Care Flashcards

1
Q

What intervention will the provider implement when prescribing medications to an 80-year-old patient?

A Beginning with higher doses and decrease according to the patient’s response

B Consulting the Beers list to help identify potentially problematic drugs

C Ensuring that the patient does not take more than five concurrent medications

D Reviewing all patient medications at the annual health maintenance visit

A

B Consulting the Beers list to help identify potentially problematic drugs

The Beers list provides a list of potentially inappropriate medications in all patients aged 65 and older and helps minimize drug-related problems in this age group. Older patients should be started on lower doses with gradual increase of doses depending on response and side effects. Patients who take five or more drugs are at increased risk for problems of polypharmacy, but many will need to take more than five drugs; providers must monitor their response more closely. Medications should be reviewed at all visits, not just annually.

How well did you know this?
1
Not at all
2
3
4
5
Perfectly
2
Q

What is the neurological exam in the sequence recommended by the American Academy of Neurology

A
mental status 
cranial nerves
motor system
sensory system
reflexes
How well did you know this?
1
Not at all
2
3
4
5
Perfectly
3
Q

Which of the following drugs are NOT associated with sensorineural hearing loss?

A Ibuprofen

B Aminoglycosides

C Furosemide

D Magnesium salicylate

E Acetaminophen

A

E Acetaminophen

All of these drugs except for acetaminophen can cause hearing loss. Platinum-based chemotherapeutics, aminoglycoside antibiotics, and loop diuretics have been associated with hearing loss, as have salicylates (e.g., aspirin) and some of the other NSAIDs (e.g., ibuprofen and diflunisal) and chloroquine. This list is obviously not exhaustive.

How well did you know this?
1
Not at all
2
3
4
5
Perfectly
4
Q

What does the mnemonic FAST stand for?

A Face drooping, arm weakness, speech difficulty, time to call

B Face tingling, ataxia, slurred speech, time to call 911

C Facial paralysis, arthralgia, speech pathology, tinnitus

D Facial weakness, arm drifting, speech disabled, tetany

A

A Face drooping, arm weakness, speech difficulty, time to call

AHA/ASA Stroke warning signs and symptoms. FAST. Face drooping, arm weakness, speech difficulty, and time to call 911.

How well did you know this?
1
Not at all
2
3
4
5
Perfectly
5
Q

Which of the following is NOT a common feature of idiopathic Parkinson’s disease?

A Rigidity

B Extraocular movement paresis

C Bradykinesia

D Postural instability

E Asymmetric resting tremor

A

B Extraocular movement paresis

There are four cardinal features of Parkinson disease: tremor, bradykinesia, rigidity, and postural instability. Two or more of these features should be present to make the diagnosis. The tremor of Parkinsonism is classically a resting tremor (as opposed to the postural, intention, or action tremor) and is most common in the hands, typically with one side affected more than another. Rigidity (“A”) is described as increased resistance to passive movement. Cogwheel rigidity is a ratchet-like sensation noted when testing a limb with concurrent tremor. Extraocular movement paresis (“B”) is more commonly seen in progressive supranuclear palsy (PSP). Bradykinesia (“C”) may be observed by monitoring the speed and amplitude of movements. Gait disturbance (“D”) with reduced stride length and stooped posture is a common finding, but generally occurs later in the course of the disease. Postural stability and ability to rise from a chair are also impaired. Postural stability may be tested by retropulsion.

How well did you know this?
1
Not at all
2
3
4
5
Perfectly
6
Q

The practitioner is establishing a plan for routine health maintenance for a new client who is 80 years old. The client has never smoked and has been in good health. What will the practitioner include in routine care for this patient? (Select all that apply.)

A Annual hypertension screening

B Baseline abdominal aorta ultrasound

C Colonoscopy every 10 years

D One-time hepatitis B vaccine

E Pneumovax vaccine if not previously given

F Yearly influenza vaccine

A

E Pneumovax vaccine if not previously given

F Yearly influenza vaccine

For older clients a one-time pneumovax is given after age 65. Influenza vaccine should be given every year. Hypertension screening should be performed at each office visit, not just annually. An abdominal aorta US is performed once for every smoking male. Colonoscopy is performed every 10 years after age 50, but not after age 74.

How well did you know this?
1
Not at all
2
3
4
5
Perfectly
7
Q

A 70-year-old male presents to your office as a new patient. He is with his wife, who assists in providing the history. His appetite is reduced, and he has lost 10 pounds in the past six months. His only medication is aspirin, and he has no significant past medical history. On examination, his vital signs are normal, and he is in no acute distress. His gait is slow, and he takes eight steps to turn. He has retropulsion (takes two steps backward when you pull him from behind). There is a resting tremor in both hands but more prominently in the right. You find cogwheel rigidity in both arms as well, but again more prominently displayed on the right. His cognitive screening tests are normal. What is the diagnosis

A Essential tremor

B Parkinson’s disease

C Normal pressure hydrocephalus

D Progressive supranuclear palsy

E Stroke

A

B Parkinson’s disease

How well did you know this?
1
Not at all
2
3
4
5
Perfectly
8
Q

An 80-year-old patient becomes apathetic, with decreased alertness and a slowing of speech several days after hip replacement surgery, alternating with long periods of lucidity. What is the most likely cause of these symptoms?

A Anesthesia effects

B Delirium

C Pain medications

D Stroke

A

B Delirium

An acute presentation of these symptoms is most likely delirium since they alternate with lucid periods. The other causes may contribute to delirium by intensifying it.

How well did you know this?
1
Not at all
2
3
4
5
Perfectly
9
Q

A patient with Alzheimer’s disease (AD) is taking donepezil to treat cognitive symptoms. The patient’s son reports noting increased social withdrawal and sleep impairment. What is the initial step to manage these symptoms?

A Encourage activity and exercise

B Prescribe a selective serotonin reuptake inhibitor (SSRI)

C Recommend risperidone

D Referral to a neurologist for evaluation

A

A Encourage activity and exercise

Patients with AD may have improvement in depression with nonpharmacologic management, including exercise and increased activity. If this is not effective, an SSRI may be prescribed. Risperidone and other antipsychotics should not be prescribed.

How well did you know this?
1
Not at all
2
3
4
5
Perfectly
10
Q

An elderly patient is brought to the primary care provider after being found on the floor after a fall. The patient has unilateral sagging of the face, marked slurring of the speech, and paralysis on one side of the body. The patient’s blood pressure is 220/190 mm Hg. What is the likely treatment for this patient?

A Direct the patient to the emergency department

B Provide a referral for a neurology consult

C Monitor the patient in the office until symptoms subside

D Administer ASA in the clinic and schedule a same day MRI

A

A Direct the patient to the emergency department

The patient is exhibiting symptoms of a stroke. He should be directed to the emergency department for further evaluation.

How well did you know this?
1
Not at all
2
3
4
5
Perfectly
11
Q
What is the most common cause of
injuries, the leading cause of hospital
admissions for trauma, and the second
leading cause of injury-related deaths for all
age groups?

A. Motor vehicle accidents

B. Falls

C. Bicycle or motorcycle accidents

D. Rollerblading and roller skating accidents

A

B. Falls

Falls are the most common cause of injuries, the
leading cause of hospital admissions for trauma, and
the second-most common cause of injury-related
deaths for all age groups (about 12,000 annually).
About 1 out of 20 persons receives emergency
department care for injuries sustained in falls.
Children typically fall from buildings or other
elevated structures, whereas older adults are more
likely to fall during normal household activities.
Adult falls are usually a result of gait instability,
decreased proprioception and muscle strength,
or vision problems. Falls cause nearly 90% of all
fractures among older adults. Motor vehicle and
bicycle accidents are a cause of death in 1 out of
6 school-age children.

How well did you know this?
1
Not at all
2
3
4
5
Perfectly
12
Q

Harold, who is 77, complains that he
can ’ t hear as well as he once used to and is
reluctant to go to his weekly card game for fear
of not hearing the conversation fully. You advise
him to

A. go to your card games, but sit so that you can read
the lips of other players.

B. see an audiologist and get fitted for a hearing aid.

C. not bother to do anything because this type of
hearing loss seen with aging is normal and can ’ t
be corrected.

D. skip the card game and take up reading because it
is more stimulating.

A

B. see an audiologist and get fitted for a hearing aid.

Sensory changes and resulting problems can be a
major contributing factor for loss of independence
and change in lifestyle to the elderly. Loss of
hearing can put your elderly client at risk for
accidents and can create social isolation. Assisting
your elderly client with referrals and the utilization
of assistive devices is an important service to your
client.

How well did you know this?
1
Not at all
2
3
4
5
Perfectly
13
Q

Your 73-year-old newly diagnosed
Alzheimer ’ s client comes in for an appointment
with her daughter. The daughter asks if there
is anything she and her family can do to help
the patient. Your best advice is which of the
following?

A. “ Try to give your mother several new stimulating
skills to perform. ”

B. “ Assist your mother in and monitor her ability
to perform activities of daily living (ADLs), and
maintain a safe environment. ”

C. “ There is little we can do to slow the progression
of this disease. ”

D. “ Your mother should be enrolled in a day-care
facility for Alzheimer ’ s patients. ”

A

B. “ Assist your mother in and monitor her ability
to perform activities of daily living (ADLs), and
maintain a safe environment. ”

Maintaining safety and preserving the ability to
perform ADLs are paramount in the patient with
cognitive issues. Because this client has already
been diagnosed, and because it is known that
memory loss is one of the first signs of Alzheimer ’ s
disease, teaching new skills could be overwhelming
and contribute to the sense of frustration that
these patients often feel. Some interventions that
can slow the progression include medications
such as Aricept in the early stages of the disease,
ADL training, and therapeutic recreational
activities. There are excellent day-care facilities for
Alzheimer ’ s patients, which may — not necessarily
should — be considered in the future as a form of
respite and assistance to clients as needed.

How well did you know this?
1
Not at all
2
3
4
5
Perfectly
14
Q

Mr. Green is a vigorous 70-year-old who
comes for early assessment of dementia. He wants
to “ work ”to keep up his mental capacities. You
counsel that he should

A. make sure he gets enough rest because cells need
time to regenerate as a result of the stress of the
aging process.

B. begin taking a calcium supplement.

C. consider a hobby that challenges his mental
capacity, like building model ships or airplanes.

D. play bridge (or any group card game) several
times a week.

A

D. play bridge (or any group card game) several
times a week.

Regular interaction with others exercises social and
language skills, and playing a card game like bridge
reinforces memory, providing a form of cognitive
“ exercise. ” Along those same lines, doing crossword
puzzles and jigsaw puzzles helps exercise the mind.
The benefits of video games and simulations are
being researched at present. Although working
on model ships or airplanes may provide some
stimulation, the solitary nature of these hobbies
over time makes them not as beneficial as an
activity like card playing that demands social
interaction in addition to mental effort. Engaging in
rigorous physical activity, not resting, is considered
protective of mental abilities. An older adult would
do better to take a daily multivitamin, not just
calcium, as a mental protective strategy. Research
has also shown that a longer education as a
youth is a protective factor. Maintaining a sense
of self-efficacy — the belief, faith, and action that
“ I can do it ” — and a “ use it or lose it ” approach are
keys to effective mental functioning in older age.

How well did you know this?
1
Not at all
2
3
4
5
Perfectly
15
Q

Your female patient, age 89, lives alone. When talking about personal safety activities, you tell her

a. Wear your slippers at all times and don’t walk barefoot.
b. Wear your reading glasses when walking around so you’ll have them when you need them.
c. When smoking in bed, be sure to turn on the light to keep you awake.
d. Wear wide-base, low heel shoes with corrugated soles to help prevent slips and falls.

A

d. Wear wide-base, low heel shoes with corrugated soles to help prevent slips and falls.

How well did you know this?
1
Not at all
2
3
4
5
Perfectly
16
Q

A patient is caring for their 83 year old father at their home. The patient’s father has dementia and is unsteady on his feet. You recommend that your patient

a. Put their father in a nursing home so that they can have a life of their own
b. Take in another elderly person so that her father can have company
c. Get information on home safety and community resources
d. Lock the father’s bedroom door at night so that he will not wander into the street.

A

c. Get information on home safety and community resources

How well did you know this?
1
Not at all
2
3
4
5
Perfectly
17
Q

Jan’s mother has Alzheimer’s disease. She tells you that her mother’s recent memory is poor and that she is easily disoriented, incorrectly identifies people, and is lethargic. Jan asks you, “Is this as bad as it gets?” You tell her that her mother is in which stage of the disease?

A. Stage 1

B. Stage 2

C. Stage 3

D. Stage 4

A

C. Stage 3

Families of persons with Alzheimer’s disease (AD) need to know that AD is a progressive disorder of the brain affecting memory, thought, and language. Although the progression of the stages is individual and changes may occur rapidly or slowly over the course of several years, knowing what stage a family member is in helps family members in planning and knowing what to expect. Stage 1 is the onset, which is insidious. Spontaneity, energy, and initiative are decreased; slowness is increased; word finding is difficult; the person angers more easily; and familiarity is sought and preferred. In stage 2, supervision with detailed activities such as banking is needed, speech and understanding are much slower, and the train of thought is lost. In stage 3, personality change is marked and depression may occur. Directions must be specific and repeated for safety, recent memory is poor, disorientation occurs easily, people are incorrectly identified, and the person may be lethargic. In stage 4, apathy is noticeable. Memory is poor or absent, urinary incontinence is present, individuals are not recognized, and the person should not be alone.

How well did you know this?
1
Not at all
2
3
4
5
Perfectly
18
Q

Performing range-of-motion exercises on a client who has had a cerebrovascular accident (CVA) is an example of which level of prevention?

A. Primary prevention

B. Secondary prevention

C. Complications prevention

D. Rehabilitation prevention

A

D. Rehabilitation prevention

Performing range-of-motion exercises on a client who has had a cerebrovascular accident (CVA) is an example of rehabilitation prevention. Primary prevention would be eating a healthy diet as a young adult to prevent atherosclerosis, which might precipitate a CVA. Secondary prevention would include taking lipid-lowering drugs to prevent a CVA after having already developed hyperlipidemia. Although it is desirable to prevent any complications from the CVA, there is no level of prevention called complications prevention.

How well did you know this?
1
Not at all
2
3
4
5
Perfectly
19
Q

The two main causes of death among U.S. adults aged 65 years or older are

A. heart disease and stroke.

B. stroke and suicide.

C. heart disease and Alzheimer’s disease.

D. heart disease and cancer.

A

D. heart disease and cancer.

The two main causes of death among U.S. adults aged 65 years or older are heart disease (28.2%) and cancer (22.2%). Stroke accounts for 6.6%, chronic lower respiratory disease 6.2%, Alzheimer’s disease 4.2%, and diabetes 2.9%.

How well did you know this?
1
Not at all
2
3
4
5
Perfectly
20
Q

Who is the most important source of social support for an adult?

A. Spouse (if applicable)

B. Parents

C. Close friends

D. Children

A

A. Spouse (if applicable)

The spouse has been shown to be the most important source of social support for an adult. If there is no spouse, family members are the next most important source. Research has shown that support from outside the family cannot compensate for what is missing within the family.

How well did you know this?
1
Not at all
2
3
4
5
Perfectly
21
Q

A lab value that is commonly decreased in older adults is

A. creatinine clearance.

B. serum cholesterol.

C. serum triglyceride.

D. blood urea nitrogen.

A

A. creatinine clearance.

The creatinine clearance value is commonly decreased in older adults because of impaired renal function. Serum cholesterol, serum triglyceride, and blood urea nitrogen values are usually increased in older adults.

How well did you know this?
1
Not at all
2
3
4
5
Perfectly
22
Q

Many of the 78 million baby boomers have a hearing loss. In a survey, all of the following statements were reported by baby boomers. Which statement was shared by the greatest percentage?

A. Hearing loss is affecting the home life of baby boomers.

B. Baby boomers have problems hearing on cell phones.

C. Baby boomers are reluctant to admit the impact of their hearing loss.

D. Hearing loss is affecting the work/jobs of baby boomers

A

C. Baby boomers are reluctant to admit the impact of their hearing loss.

When the 78 million baby boomers were growing up, television, rock concerts, and other intense audio programs were coming of age. Earplugs were unheard of. This generation does not want to wear hearing aids or anything that marks them as being different or disabled. As a result, baby boomers frequently avoid seeking help from hearing aid professionals. The following statistics are from a national survey of baby boomers: 75% said they find themselves in situations in which people are not speaking loudly or clearly enough or they can’t hear the TV, 53% said they have at least a mild hearing loss, 25% said their hearing loss affects their work, and 57% said they have trouble hearing on their cell phone.

How well did you know this?
1
Not at all
2
3
4
5
Perfectly
23
Q

Oral health problems are common and painful. Years ago, it was common to see almost all adults with complete dentures. Today, what percentage of U.S. adults aged 75 and older have lost all of their teeth?

A. 10%

B. 25%

C. 50%

D. 75%

A

B. 25%

Twenty-five percent of U.S. adults aged 75 and older have lost all of their teeth. Advanced gum disease affects 4%–12% of adults. Half of the cases of severe gum disease in the United States are the result of cigarette smoking. Three times as many smokers have gum disease as people who have never smoked. More than 7,600 people, mostly older Americans, die from oral and pharyngeal cancers each year.

How well did you know this?
1
Not at all
2
3
4
5
Perfectly
24
Q

Sleep in older adults is characterized by which pattern?

A. Increased time spent in REM sleep

B. Increased overall sleep time

C. Increased sleep latency

D. Increased proportion of deep sleep

A

C. Increased sleep latency

Older adults typically have prolonged sleep latency or longer period of time between going to bed and falling asleep. Other sleep characteristics associated with aging include more frequent nighttime awakening, less REM sleep, less deep sleep, earlier awakenings, and less overall sleep time.

How well did you know this?
1
Not at all
2
3
4
5
Perfectly
25
Q

Older adults face greater risks for fluid imbalance than young and middle adults due to which age-related factor?

A. Increased amounts of intracellular fluid and total body water

B. Higher proportion of fat to muscle cells

C. Faster speed of metabolism

D. Increased intestinal motility

A

B. Higher proportion of fat to muscle cells

Older adults ’ bodies have fewer muscle cells and more fat cells than those who are younger. Fat cells contain less fluid than muscle cells; thus the intracellular compartment has less fluid. Both total body water and intracellular fluid volumes are lower than in younger individuals, placing the older adult at higher risk for fluid imbalance.

How well did you know this?
1
Not at all
2
3
4
5
Perfectly
26
Q

Evaluating an older adult client according to the successful aging paradigm involves assessment of

A. cognition, problem-solving, physical skills, and memory.

B. health, social engagement, mental activity, and life satisfaction.

C. vision, hearing, balance, and strength.

D. coping, social support, financial resources, and living arrangement

A

B. health, social engagement, mental activity, and life satisfaction.

The successful aging paradigm suggests that physical health, stimulation of cognitive capabilities, social involvement and engagement, productivity, and life satisfaction are major components of a healthy aging process. Assessment of these components can help the clinician understand an individual ’ s aging process, typical patterns of daily living, and unmet needs.

How well did you know this?
1
Not at all
2
3
4
5
Perfectly
27
Q

Which of the following is a result of changes in the cell-mediated immune function that occur from aging?

A. Fewer neutrophils, so older adults do not respond to local infections quickly

B. Increases in autoantibodies as a result of altered immune function

C. Fewer thrombocytes

D. The pancytopenia of aging

A

B. Increases in autoantibodies as a result of altered immune function

Due to the change in function of T and B lymphocytes in older adults, the formation of antibodies shifts to form antibodies against a person ’ s own immune system. Normal aging does not in and of itself cause a decrease in thrombocytes or the other cell lines.

How well did you know this?
1
Not at all
2
3
4
5
Perfectly
28
Q

Photosensitivity of the epidermis increases in older adults due to which of the following normal developmental changes of aging skin?

A. Reduction of Langerhans cells

B. Decrease in vitamin D synthesis

C. Decrease in enzymatically active melanocytes

D. Significant decrease in vascular supply

A

C. Decrease in enzymatically active melanocytes

Increased sensitivity to sunlight exposure increases in older adults as the number of melanocytes (pigment-carrying cells) decreases. This results in graying hair, reduced capillary blood supply, and fading skin color. Langerhans cells do decrease by 40%, but this is generally assumed to account for the increased hypersensitivity of skin in older adults, not increased photosensitivity.

How well did you know this?
1
Not at all
2
3
4
5
Perfectly
29
Q

Physiologic changes of aging can affect functional mobility. Screening for functional mobility of older adults involves which screening tool?

A. Katz Index

B. Get Up and Go Test

C. Functional Independence Measure

D. Mini Mental State Exam

A

B. Get Up and Go Test

The Get Up and Go Test is a structured observation to evaluate functional mobility. To assess a patient, have the person rise from a chair, walk 10 feet forward, turn, and return to the chair and sit down again. Strength, balance, gait, pace, and coordination in turning are evaluated by the observer.

How well did you know this?
1
Not at all
2
3
4
5
Perfectly
30
Q

What is the major determinant of sexual activity in older adults?

A. Physical and mental health

B. Social sanctions

C. Religious beliefs

D. Age

A

A. Physical and mental health

Evidence suggests that people are sexually active well into their 80s and 90s. The two major factors determining sexual activity are good physical and mental health and previous regular sexual activity. The phrase “ use it or lose it ” seems to be relevant.

How well did you know this?
1
Not at all
2
3
4
5
Perfectly
31
Q

Which strategy for care of older adults is inconsistent with an approach supportive of aging in place?

A. Make changes to the home environment that can accommodate an individual ’ s changing needs.

B. Refer for home care and adult day-care services.

C. Hire assistants to help with activities of daily living.

D. Emphasize the individual ’ s limitations and likely need for long-term care placement.

A

D. Emphasize the individual ’ s limitations and likely need for long-term care placement.

Aging in place involves modification of the home environment and provision of support to keep aging adults in their homes. Research demonstrates that most older adults wish to remain in their homes and die at home.

How well did you know this?
1
Not at all
2
3
4
5
Perfectly
32
Q

Which of these musculoskeletal changes accompany older adulthood?

A. Change in stature

B. Increased stride length

C. Improved fine motor skill

D. Stretching of muscle fibers

A

A. Change in stature

Aging is accompanied by decreased muscle strength, decreased endurance, loss of flexibility and connective tissue elasticity, decreased bone mass, and decreases in stature due to changes in spine architecture.

How well did you know this?
1
Not at all
2
3
4
5
Perfectly
33
Q

Why is Alzheimer ’ s disease (AD) considered a specific disease, distinct from the normal developmental changes of aging?

A. There are types of AD that are inherited and present before age 60.

B. There are changes in biological processes that occur with normal aging.

C. There is a notable decline in cognitive functioning as people age.

D. There are multiple changes in organ functioning, especially in the brain.

A

A. There are types of AD that are inherited and present before age 60.

Some types of AD are inherited and present before age 60. There are also other forms of dementia, such as Pick ’ s disease, that have the same outcome as AD but can be found as early as the fifth decade. These diseases have a genetic predisposition and may not be a result of the aging process. It should be remembered, however, that half of all people older than 85 are affected by AD.

How well did you know this?
1
Not at all
2
3
4
5
Perfectly
34
Q

When performing a mental status examination, you should keep in mind the ABCTs. What does the C stand for?

A. Cognition

B. Consciousness

C. Capabilities

D. Conscience

A

A. Cognition

The ABCTs are the four main headings of a mental status assessment. The A stands for appearance, the B for behavior, the C for cognition, and the T for thought processes.

How well did you know this?
1
Not at all
2
3
4
5
Perfectly
35
Q

A 75-year-old man comes to your office for concerns about memory loss. He is worried that he has Alzheimer ’ s disease. Which finding in the patient would be cause for concern?

A. Often misplaces his car and house keys

B. Has difficulty remembering some of the names of people he has recently met

C. Sometimes forgets the names of his children

D. Recalls the names of only seven of the nine medications he is currently taking

A

C. Sometimes forgets the names of his children

Some difficulty with recall is an expected agerelated cognitive change. However, impaired recall of common words or loved one ’ s names may indicate significant decline associated with dementia, including Alzheimer ’ s disease. Further tests of cognitive function and referral to neurology or geriatric specialist are indicated

How well did you know this?
1
Not at all
2
3
4
5
Perfectly
36
Q

When completing the health history and review of systems for a healthy 88-year-old woman, you would expect which age-related change to be reported?

A. Mildly blurry vision

B. Chronically dry and itchy eyes and eyelids

C. Increasing presbyopia

D. Tearing and photophobia

A

C. Increasing presbyopia

Worsening near vision, or presbyopia, is an expected age-related change. Blurring may indicate the development of cataracts. Chronic itching and tearing, or photophobia, can be related to many disorders.

How well did you know this?
1
Not at all
2
3
4
5
Perfectly
37
Q

Which cognitive change is expected in healthy older adults age 65 and older?

A. Decrease in IQ

B. Slower information processing

C. Low capacity for learning

D. Decreased attentional focus

A

B. Slower information processing

Slowing of processing and accessing information is a cognitive change associated with aging.

How well did you know this?
1
Not at all
2
3
4
5
Perfectly
38
Q

What is the first symptom seen in the majority of patients with Parkinson’s dx?

A. Rigidity

B. Bradykinesia

C. Rest tremor

D. Flexed posture

A

C. Rest tremor

Onset of motor signs in Parkinson dx is typically asymmetric, with the most common initial finding being an asymmetric resting tremor in an upper extremity. Over time, patients notice symptoms related to progressive bradykinesia, rigidity, and gait difficulty.

How well did you know this?
1
Not at all
2
3
4
5
Perfectly
39
Q

the older adult with delirium would present with which of the following behaviors?

A. Fatigue, apathy, and occasional agitation

B. Agitation, apathy, and wandering behavior

C. Agitation and restlessness

D. Slowness and absence of purpose

A

C. Agitation and restlessness

The older adult with delirium would present with agitated and restless behavior.

How well did you know this?
1
Not at all
2
3
4
5
Perfectly
40
Q

During a mental status exam, which question would be the most helpful to assess remote memory?

A. “How long have you been here?”

B. “What time did you get here today?”

C. “What did you eat for breakfast?”

D. “What was your mother’s maiden name?”

A

D. “What was your mother’s maiden name?”

Remote memory is verbalized after hours, days, or years and may be assessed by asking a patient his or her mother’s maiden name. Asking questions about things that happened today, such as how long the patient has been at your office, what time the patient arrived, and what he or she ate for breakfast, tests recent memory.

How well did you know this?
1
Not at all
2
3
4
5
Perfectly
41
Q

What is the main overall goal of therapy for the patient with Parkinson’s dx?

A. The halt the progression of the dx

B. To keep the patient functioning independently as long as possible

C. To control the symptoms of the dx

D. To ease the depression associated with the dx

A

B. To keep the patient functioning independently as long as possible

The main overall goal of therapy for the patient with Parkinson’s disease is to keep the patient functioning independently for as long as possible. There is no drug or surgical approach that will prevent the progression of the disease. Treatment is aimed at controlling symptoms. Depression occurs in more than 50% of patients with Parkinson’s disease, and it is undetermined whether it is a reaction to the illness or a part of the illness itself.

How well did you know this?
1
Not at all
2
3
4
5
Perfectly
42
Q

Julie, age 58, has had several TIAs. After a diagnostic evaluation, what medication would you start her one?

A. Ticlopidine (Ticlid)

B. Clopidogrel (Plavix)

C. Warfarin (Coumadin)

D. Nitroglycerin

A

B. Clopidogrel (Plavix)

Clopidogrel (Plavix) an antiplatelet drug used to reduce blood clot formation by preventing platelets from “sticking” together and forming blood clots, should be ordered. The safety of Plavix is comparable to that of aspirin (ASA) and it has clear advantages over ticlopidine. Since ASA is not listed as an answer, clopidogrel is the choice. As with ticlopidine, diarrhea and rash are more frequent than with ASA, but GI symptoms and hemorrhages are less frequent. For patients intolerant to ASA because of an allergy or GI side effects, clopidogrel is an appropriate choice. The benefit of ASA in women has not been clearly proved. ASA also has more side effects than ticlopidine. The desired effect is decreased platelet aggregation rather than anticoagulation, which warfarin would accomplish. Nitroglycerin has no effect on platelets.

How well did you know this?
1
Not at all
2
3
4
5
Perfectly
43
Q

Lynne, age 72, presents for the first time with her daughter. Her daughter describes some recent disturbing facts about her mother. How can you differentiate between depression and dementia?

A. You might be able to pinpoint the onset of dementia, but the onset of depression is difficult to identify

B. A depressed person has wide mood swings, whereas a person with dementia demonstrates apathetic behavior

C. The person with dementia tries to hide problems concerning his or her memory, whereas the person with depression complains about memory

D. The person with dementia has a poor self-image, whereas the person with depression does not have a change in self-image

A

C. The person with dementia tries to hide problems concerning his or her memory, whereas the person with depression complains about memory

To help differentiate between depression and dementia, keep in mind that the person with dementia tries to hide problems concerning memory, whereas the person with depression complains about memory and discusses the fact that there is a problem with memory. Also, with depression there is usually a time-specific onset, and affected clients tend to be apathetic and withdrawn and have a poor self-mage.

How well did you know this?
1
Not at all
2
3
4
5
Perfectly
44
Q

Paul has recently been diagnosed with Parkinson’s dx and is beginning a new dru regimen that includes carbidopa/levodopa (Sinemet). Important medication instructions to review with the patient and his family include which of the following?

A. It is important to take the medication at the dose and times directed without adjustment

B. The medication is to be taken with food

C. The medication is to be taken only on an empty stomach

D. If a dose is missed, it is important to add it to the next scheduled dose

A

A. It is important to take the medication at the dose and times directed without adjustment

Carbidopa levodopa (Sinemet) is prescribed with a tightly regimented schedule. Missed, additional, or poorly timed doses have adverse effects over te course of the Parkinson’s patient’s life – including worsening of the disease and its side effects, Sinemet may be taken with or without food.

How well did you know this?
1
Not at all
2
3
4
5
Perfectly
45
Q

Which of the following symptoms related to memory indicates depression rather than delirium or dementia in the older adult?

A. Inability to concentrate, with psychomotor agitation or retardation

B. Impaired memory, especially of recent events

C. Inability to learn new material

D. Difficulty with long-term memory

A

A. Inability to concentrate, with psychomotor agitation or retardation

The prevalence of depression (5%-10%) does not change with age, but depression is often overlooked in the older adult. The diagnosis requires a depressed mood for 2 weeks and at least for of the following 8 signs, which can be remembered using the mnemonic SIG E CAPS (as when prescribing energy caps): S = sleep disturbance; I = lack of interest; G – feelings of guilt; C = decreased concentration; A = decreased appetite; P = psychomotor agitation or retardation, and S = suicidal ideation.
Dementia and delirium often coexist with depression. Delirium is a confusional state characterized by inattention, rapid onset, and a fluctuating course that may persist months if untreated. The person with delirium has memory impairment, such as inability to learn new material or to remember past events. With dementia, there is a cognitive deterioration with little or no disturbance of consciousness or perception; attention span and short-term memory are impaired; along with judgement, insight, spatial perception, abstract reasoning, and thought process and content.

How well did you know this?
1
Not at all
2
3
4
5
Perfectly
46
Q

Which appropriate test for the initial assessment of Alzheimer’s dx provides performance ratings on 10 complex, higher-order activities?

A. MMSE

B. CAGE questionnaire

C. FAQ

D. Holmes and Rahe Social Readjustment scale

A

C. FAQ

The FAQ (Functional Activities Questionnaire) is a measure of functional activities. There are 10 complex, higher-order activities that are appropriate for the initial assessment of Alzheimer’s dx. The MMSE is a test of cognition. The CAGE is a screening tool for alcoholism. The Holmes and Rahe Social readjustment scale measures major life changes for identifying the impact of stress on an individual.

How well did you know this?
1
Not at all
2
3
4
5
Perfectly
47
Q

Sally, age 81, presents to the clinic at 9am, with her daughter, who reports her mother seems confused and is having trouble speaking. Sally presents with right facial droop and expressive aphasia, prompting the staff to call for transport to the nearby comprehensive stroke hospital. In reporting the status of the patient to the receiving hospital, you are aware that which od the following questions is most important in providing her treatment immediately upon arrival to the ED?

A. When was the last time she ate?

B. Has this ever happened to the patient before?

C. When was the last time she was seen normal?

D. Is she able to swallow?

A

C. When was the last time she was seen normal?

Because the patient presents early in the day, is is possible that she awoke with the symptoms, which will mean she is not eligible for emergent rescue with tPA. However, if the daughter reports that Sally was walking and talking during an early breakfast, her “last normal” falls within the guidelines of less than 3 hours since time of onset. Treatment with tPA does not require the patient to be NPO. Prior stroke is an important hx to report, but will not preclude the patent from receiving tPA. A dysphagia screen is routine assessment of stroke patients and is completed in the ED prior to admission of any oral meds.

How well did you know this?
1
Not at all
2
3
4
5
Perfectly
48
Q

During a mental status exam, which question might you ask to assess abstraction ability?

A. “What does a “rolling stone gathers no moss” means?”

B. “Start with 100 and keep subtracting 7”

C. “What do you think is the best treatment for your problem?”

D. “What would you do if you were in a restaurant and a fire broke out?”

A

A. “What does a “rolling stone gathers no moss” means?”

Asking the client the meaning of a familiar proverb assesses the abstraction ability. It must be kept in mind that many proverbs are culturally derived, and the client may not have heard them before. Asking the client to subtract numbers tests computational ability. Asking clients what they think is the best tx for their problem elicits information about their mental representation or beliefs about the illness. Asking clients what they would do if a fire broke out in a restaurant assesses their judgment.

How well did you know this?
1
Not at all
2
3
4
5
Perfectly
49
Q

Sophie, a retired English professor, is 82 and scores 25 on the Mimi-Mental State Examination (MMSE). What is your initial thought?

A. Normal

B. Depression

C. Early Alzheimer’s dx

D. Late Alzheimer’s dx

A

C. Early Alzheimer’s dx

The total possible score on the MMSE is 30. The median score for persons aged 18- 59 years old is 29. For persons aged 80 and older, the median score is 25. However, with Sophie’s education level there may be cause for concern that she is exhibiting early signs of dementia. A score of 20 to 25 indicates early Alzheimer’s dx, a score of 10 to 19 indicates middle-stage Alzheimer’s dx. Someone with late-stage Alzheimer’s dx may score below 10.

How well did you know this?
1
Not at all
2
3
4
5
Perfectly
50
Q

Dave, age 76 is brought in by his wife, who states that within the past 2 days Dave has become agitated and restless, has had few lucid moments, slept very poorly last night, and can remember only recent events. Of the following differentials, which seems the most logical from this brief history?

A. Depression

B. Dementia

C. Delirium

D. Schizophrenia

A

C. Delirium

The DSM-5 lists 5 key features that characterize delirium: (1) disturbance in attention (reduced ability to direct, focus, sustain, and shift attention) and awareness; (2) the disturbance develops over a short period of time (usually hours to days), represents a change from baseline, and tends to fluctuate during the course of the day; (3) an additional disturbance in cognition (memory deficit, disorientation, language, visuospatial ability, or perception); (4) the disturbances are not better explained by another preexisting, evolving, or established neurocognitive disorder, and do not occur in the context of a severely reduced level of arousal, such as coma; (5) there is evidence from the hx, PE, or labs that the disturbance is caused by a medical condition, substance intoxification or withdrawal, or medication side effect

How well did you know this?
1
Not at all
2
3
4
5
Perfectly
51
Q

Which of these neurological findings is an early sign of Parkinson’s dx?

A. Hemianopsia

B. Anosmia

C. Nystagmus

D. Babinski sign

A

B. Anosmia

Anosmia, the loss of smell, is a common nonmotor feature of Parkinson dx. Ageusia, the loss of taste, is also an underappreciated nonmotor feature of Parkinson dx. The olfactory tract is involved early in Parkinson dx as indicated by frequent occurrence of hyposmia or anosmia years or decades before motor symptoms.

How well did you know this?
1
Not at all
2
3
4
5
Perfectly
52
Q

Which of the following drugs used for parkinsonism mimics dopamine?

A. Anticholinergics

B. Levodopa

C. Bromocriptine

D. Tolcapone

A

C. Bromocriptine

Bromocriptine and pergolide mimic dopamine. The other mechanisms of antiparkinsonian treatments are as follows: Anticholinergics restore acetylcholine-dopamine balance; levodopa restores striatal dopamine; and tolcapone and entacapone reduce systemic degradation of oral dopamine.

How well did you know this?
1
Not at all
2
3
4
5
Perfectly
53
Q

When you ask a client to walk a straight line, placing heel to toe, you are assessing

A. Sensory function

B. Cerebellar function

C. Cranial nerve function

D. Proprioceptive function

A

D. Proprioceptive function

When you ask a patient to walk a straight line placing heel to toe, you are assessing the proprioceptive aspect of the nervous system, which control posture, balance, and coordination. Assessing dermatomes and the major peripheral nerves tests sensory function. The cerebellum is one of the neural structures involved in proprioception and can be assessed in many ways, but not by walking.

How well did you know this?
1
Not at all
2
3
4
5
Perfectly
54
Q

A video swallow is ordered for James, a patient with senile dementia of the Alzheimer’s type, because he is at risk for which complication that often results in death?

a. Pneumonia
b. Delirium
c. Aphasia
d. Hyperthyroidism

A

a. Pneumonia

How well did you know this?
1
Not at all
2
3
4
5
Perfectly
55
Q

Which of the following interventions can significantly slow the decline in performing ADLs in clients with Alzheimer’s dx living in a nursing home?

A. A simple exercise program

B. Ginkgo biloba

C. Doing crossword puzzles

D. Improving nutritional state

A

A. A simple exercise program

A simple exercise program, 1 hour twice a week has shown to significantly slower decline in ADLs in patients.

How well did you know this?
1
Not at all
2
3
4
5
Perfectly
56
Q

How common is stroke in the United States population?

A

Stroke is the fifth leading cause of death and a leading cause of disability among adults.

It is one of the most common reasons for admission to the hospital.

Approximately 1 out of every 20 deaths in the United States is caused by stroke. In the past 10 years rate of death from stroke however has fallen by 33%.

How well did you know this?
1
Not at all
2
3
4
5
Perfectly
57
Q

A 60-year-old female presents to establish care in the primary care clinic. Her history is significant for having had a stroke 1 month ago. The stroke caused left arm and leg clumsiness. She is able to walk on her leg, but feels it is difficult. She also was confused earlier on the day of the event and had some difficulty with speech. She has a history of migraines, but has had no recent migraine. She does not smoke. Her medication at the time of the stroke was sumatriptan as needed for migraine. Family history is significant for her mother and grandfather both having strokes in their 60s. MRI was obtained which demonstrated right thalamic stroke.

Her office weight is 180 lb, body mass index (BMI) is 29.7 kg/m2. Blood pressure is 146/90 mm Hg.

What modifiable risk factors should you address to prevent recurrent cerebrovascular events?

A
●	Hypertension
●	Obesity
●	Atrial fibrillation (AF) / cardiac disease
●	Diabetes
●	Dyslipidemia
●	Excessive alcohol intake
●	Physical inactivity
●	Smoking
●	Diet
For completeness the nonmodifiable risk factors are listed below:
●	Age
●	Gender
●	Family history
●	Ethnicity
●	Previous TIA or stroke
How well did you know this?
1
Not at all
2
3
4
5
Perfectly
58
Q

What behavioral risk factors can you review with the 60 y.o F w/ hx of stroke within past month?

A

● Diet, sodium, exercise, weight, smoking, alcohol intake
All patients should receive information and counseling about possible strategies to modify their lifestyle and risk factors.

How well did you know this?
1
Not at all
2
3
4
5
Perfectly
59
Q

The patient’s blood pressure in the office is 146/90 mm Hg. What do you think is the single most powerful risk factor for a recurrent stroke?

A

Blood pressure is the most consistent and powerful predictor of recurrent stroke such that hypertension is causally involved in nearly 70% of all stroke cases. Small reductions in blood pressure have big impacts on incidence of stroke. A reduction in blood pressure of 10/5 mm Hg will reduce the risk of stroke by 35% to 42%.

Significant reductions in recurrent stroke were seen with diuretics alone and in combination with angiotensin-converting enzyme (ACE) inhibitors but not with beta-blockers or ACE inhibitors used alone.

How well did you know this?
1
Not at all
2
3
4
5
Perfectly
60
Q

ACC/AHA HTN guidelines for those with stroke, TIA, or lacunar stroke

A

recommend a blood pressure goal of less than or equal to 130/80 mm Hg

Recommendations from American Heart Association (AHA)/ American Stroke Association (ASA)
● Blood pressure reduction is recommended for both prevention of recurrent stroke and prevention of other vascular events in persons who have had an ischemic stroke or TIA and are beyond the first 24 hours (Class I; Level of Evidence A).
● Because this benefit extends to persons with and without a documented history of hypertension, this recommendation is reasonable for all patients with ischemic stroke or TIA who are considered appropriate for blood pressure reduction (Class IIa; Level of Evidence B).
● An absolute target blood pressure level and reduction are uncertain and should be individualized, but benefit has been associated with an average reduction of approximately 10/5 mm Hg (Class IIa; Level of Evidence B).
● The optimal drug regimen to achieve the recommended level of reduction is uncertain because direct comparisons between regimens are limited. The available data indicate that diuretics or the combination of diuretics and an ACE inhibitor are useful (Class I; Level of Evidence A).
● The choice of specific drugs and targets should be individualized on the basis of pharmacological properties, mechanism of action, and consideration of specific patient characteristics for which specific agents are probably indicated (eg, extracranial cerebrovascular occlusive disease, renal impairment, cardiac disease, and diabetes) (Class IIa; Level of Evidence B).

How well did you know this?
1
Not at all
2
3
4
5
Perfectly
61
Q

A 60-year-old female presents to establish care in the primary care clinic. Her history is significant for having had a stroke 1 month ago.
Her labs from the stroke hospitalization are significant for normal thyroid function, total cholesterol 169 mg/dL, high-density lipoprotein (HDL) cholesterol 50 mg/dL, and low-density lipoprotein (LDL) of 103 mg/dL.

What should you do regarding the initial management of her dyslipidemia? Does she really need treatment since she does not have known heart disease?

A

Recommendations from AHA/ASA
● Statin therapy with intensive lipid-lowering effects is recommended to reduce risk of stroke and cardiovascular events among patients with ischemic stroke or TIA who have evidence of atherosclerosis, an LDL-C level greater than 100 mg/dL, and who are without known CHD (Class I; Level of Evidence B).
● For patients with atherosclerotic ischemic stroke or TIA and without known CHD, it is reasonable to target a reduction of at least 50% in LDL-C or a target LDL-C level of less than 70 mg/dL to obtain maximum benefit (Class IIa; Level of Evidence B). (New recommendation.)
● Patients with ischemic stroke or TIA with elevated cholesterol or comorbid coronary artery disease (CAD) should be otherwise managed according to National Cholesterol Education Program (NCEP III) guidelines, which include lifestyle modification, dietary guidelines, and medication recommendations (Class I; Level of Evidence A).

How well did you know this?
1
Not at all
2
3
4
5
Perfectly
62
Q

A 60-year-old female presents to establish care in the primary care clinic. Her history is significant for having had a stroke 1 month ago.
The patient has no carotid bruits, and her cardiac exam is normal rhythm with no murmur. Her carotids were normal on an MRA of the neck. She was scheduled for an outpatient echocardiogram, which revealed normal ejection, no patent foramen ovale (PFO) by bubble study, and sinus rhythm.

There are other common and uncommon cardiac causes of stroke. Name a few which the clinician should consider.

A
●	Cardiac cause of stroke: AF and PFO 
●	Extracardiac cause: intracranial and extracranial large vessel atherosclerotic disease (carotid artery / vertebral-basilar disease)
Atrial Fibrillation (AF): The optimal intensity of oral anticoagulation for stroke prevention in patients with AF appears to be an international normalizing ratio (INR) of 2.0 to 3.0.
How well did you know this?
1
Not at all
2
3
4
5
Perfectly
63
Q

A 60-year-old female presents to establish care in the primary care clinic. Her history is significant for having had a stroke 1 month ago.

The patient was only on sumatriptan prior to stroke. She wonders if she needs other medications now.
Construct the patient’s new suggested medication list:

A

● Aspirin
● ACE inhibitor and thiazide diuretic
● Statin
● Consider antidepressant (post-stroke depression can be severe)

How well did you know this?
1
Not at all
2
3
4
5
Perfectly
64
Q

post stroke depression

A

common consequence of stroke and reported to occur in almost 40% of patients who have had a stroke.

Post-stroke depression is associated with poor functional recovery, poor social outcomes, reduced quality of life and rehab along with increased cognitive impairment. Apathy—motivational disorder is found in 23% to 57% of patients with stroke.
Screen patient for depressive symptoms and initiate treatment as necessary—most studies have examined citalopram in post-stroke depression but selective serotonin reuptake inhibitors (SSRIs) in general seem to be effective.

How well did you know this?
1
Not at all
2
3
4
5
Perfectly
65
Q

antiplatelets vs anticoagulants for stroke

A

Recommendations from AHA/ASA
● For patients with noncardioembolic ischemic stroke or TIA, the use of antiplatelet agents rather than oral anticoagulation is recommended to reduce the risk of recurrent stroke and other cardiovascular events (Class I; Level of Evidence A).
● Aspirin (50 mg/d to 325 mg/d) monotherapy (Class I; Level of Evidence A), the combination of aspirin 25 mg and extended-release dipyridamole 200 mg twice daily (Class I; Level of Evidence B), and clopidogrel 75 mg monotherapy (Class IIa; Level of Evidence B) are all acceptable options for initial therapy.

How well did you know this?
1
Not at all
2
3
4
5
Perfectly
66
Q

A 60-year-old female presents to establish care in the primary care clinic. Her history is significant for having had a stroke 1 month ago.

The patient is working daily as a phlebotomist (on modified duty) but walks with a limp
What else should you consider for her?

A
Ask about the need for assistive devices and services. A common complication suffered by many patients who have had a stroke is falls. Do not ignore these "little spells" or "trips." These all impede the patient's functional status and can impair rehab and recovery. Providing stroke survivors with assistive devices such as walkers and canes can help them have more freedom and make the caregiver's job easier. Here are some examples of assistive devices that may be helpful:
●	Wheelchair
●	Bath brush
●	Walker
●	Shower grab bar 
●	Reacher/ gripper
●	Quad cane
●	Shirt buttoner
●	Tub bench
●	Elevated toilet seat 
●	Urinal
How well did you know this?
1
Not at all
2
3
4
5
Perfectly
67
Q

Which of the following signs and symptoms is most specific for transient ischemic attack (TIA) or stroke?

A. Memory loss
B. Headache
C. Speech impairment
D. Blurred vision

A

C. Speech impairment

There are many mimics of TIA including seizures, migraines, syncope, and metabolic disturbances such as hypoglycemia. Speech impairment is much more likely to be due to TIA compared to other causes. While memory loss, headache, and blurred vision may be associated with TIA and stroke, these symptoms are also associated with many other clinical conditions.

How well did you know this?
1
Not at all
2
3
4
5
Perfectly
68
Q

What are the most common presenting symptoms in a patient with TIA or acute stroke?

A. Unilateral paresis and dysarthria
B. Dysarthria and headache
C. Unilateral paresis and cognitive impairment
D. Dysarthria and diplopia

A

A. Unilateral paresis and dysarthria

The most common presenting symptoms in a patient with TIA are unilateral paresis (58%) and dysarthria (21%) and are more likely to be associated with acute cerebral infarct on MRI. Headache, cognitive impairment, and diplopia may also be a part of the clinical presentation of TIA but are less common.

How well did you know this?
1
Not at all
2
3
4
5
Perfectly
69
Q

Which of the following interventions has the highest relative risk reduction to prevent stroke in patients that have had a TIA?

A. Statin therapy
B. High-intesity physical therapy
C. Blood pressure reduction
D. Antiplatelet medications

A

B. High-intesity physical therapy

In patients that have had a TIA, high-intensity physical activity has a relative risk reduction of 64% in the prevention of stroke compared to inactivity. Blood pressure reduction conferred a relative risk reduction of 30% to 40%. Antiplatelet medications and statin therapies conferred a relative risk reduction of 18% to 37% and 16% to 33%, respectively.

How well did you know this?
1
Not at all
2
3
4
5
Perfectly
70
Q

Ms. D is 75 years old with well-controlled diabetes mellitus type 2 (T2DM) and hypertension (HTN) who presents for her annual exam. In reviewing her chart, you see that her mammogram, colonoscopy, bone density, and Pap are all up to date. She lives alone and her daughter lives close by. Her daughter accompanies her, and they ask what they can do to keep Ms. D healthy and independent.

List 5 domains that you would assess in this patient.

A

1) Physical Health
2) Cognition and Mental Health
3) Functional Status
4) Social and Environmental Circumstances
5) Advance Care Planning

How well did you know this?
1
Not at all
2
3
4
5
Perfectly
71
Q

Discuss how health maintenance of the geriatric patient differs from the younger patient.

A

● In addition to treating medical problems, the geriatric assessment includes screening patients for risk factors that can affect health and independence.
● Often requires a multidisciplinary team including nutrition, social work, physical therapy, and occupational therapy
● When should it be done?
● Yearly starting at age 75 if healthy
● Yearly when younger than 75 if the patients has multiple comorbidities
● After major illness requiring hospitalization
● May address specific problems during first visit then use rolling assessment over several visits and target one domain per visit
● In general, if the patient has more than 5 year life expectancy, consider age-appropriate screening.
● USPSTF has an electronic tool that gives screening recommendations by age (http://epss.ahrq.gov/ePSS/search.jsp)

How well did you know this?
1
Not at all
2
3
4
5
Perfectly
72
Q

Ms. D is 75 years old with well-controlled diabetes mellitus type 2 (T2DM) and hypertension (HTN) who presents for her annual exam. In reviewing her chart, you see that her mammogram, colonoscopy, bone density, and Pap are all up to date. She lives alone and her daughter lives close by. Her daughter accompanies her, and they ask what they can do to keep Ms. D healthy and independent.

List the components of Ms. D’s physical health that you plan to assess.

A
●	Nutrition
●	Vision
●	Hearing
●	Urinary and bowel incontinence
●	Balance problems and history of falls
●	Osteoporosis risk
●	Polypharmacy
How well did you know this?
1
Not at all
2
3
4
5
Perfectly
73
Q

List the most common vision problems in the geriatric population, and review current screening recommendations.

A
●	Presbyopia
●	Glaucoma
●	Diabetic retinopathy
●	Cataracts
●	Age-related macular degeneration
How well did you know this?
1
Not at all
2
3
4
5
Perfectly
74
Q

most common hearing problems in the geriatric population, and review current screening recommendations

A

Two-thirds of adults older than 70 have significant hearing loss, one-third of those have severe hearing loss.
● Can lead to social isolation, functional decline, poor quality of life, depressive symptoms, and cognitive deficits

Screening
● USPSTF recommends asking patient and caregivers about hearing loss
● Whisper test – Stand 3 feet away and whisper letters and numbers as screening test

● Most common cause of hearing impairment in older adults is presbycusis (multifactorial sensorineural hearing loss) and cerumen impaction (conductive hearing loss).

How well did you know this?
1
Not at all
2
3
4
5
Perfectly
75
Q

Discuss urinary incontinence in the geriatric population.

A
  1. Associated with decubitus ulcers, sepsis, renal failure, urinary tract infections, and increased mortality
  2. Psychosocial impairment – restriction of social and sexual activities, increased risk of depression
  3. Key deciding factor for nursing home placement
  4. Evaluation – fluid intake, medications, cognitive functions, mobility, urologic surgeries
  5. Screening questions
    ● Urge incontinence – Do you have a strong and sudden urge to void that makes you leak before reaching the toilet?
    ● Stress incontinence – Is your incontinence caused by coughing, sneezing, lifting, walking, or running?
How well did you know this?
1
Not at all
2
3
4
5
Perfectly
76
Q

Discuss the assessment of balance problems and fall risk in the geriatric population

A

● More than one-third of community-dwelling adults older than 65 fall once per year
● 10% of falls result in major injury
● Leading cause of hospitalization and injury-related death in people older than 75 years old
● Increases risk of nursing home placement

Risk factors for falls
●	Age older than 80 years old
●	Female
●	Lower body mass index (BMI)
●	Previous falls
●	Balance or gait impairment
●	Decreased upper or lower muscle strength
●	Visual impairment
●	Medications (more than 4 total meds or use of psychoactive med use)
●	Depression
●	Dizziness or orthostasis
●	Functional limitations
●	Urinary incontinence
●	Cognitive impairment
●	Arthritis
●	Diabetes
●	Pain

Screening
● Get Up and Go Test
● Observing a patient get up from chair without using arms, walk 10 feet, turn around, walk back, sit down (should not take longer than 16 seconds) → anything concerning needs further evaluation
● One fall without major injury and normal Get Up and Go Test
● → No further evaluation needed

Decrease fall risk with exercise, physical therapy, home hazard assessment, decreasing psychotropic medications

Home assessment and modification can significantly decrease the number of falls (can be done by family or occupational therapy).

How well did you know this?
1
Not at all
2
3
4
5
Perfectly
77
Q

Discuss the assessment of polypharmacy and an approach to medication discontinuation

A

● Beers criteria – list of medications to avoid in the elderly
● 20% of community-dwelling adults older than 65 year old take more than 10 meds
● More meds leads to:
● Increased risk of adverse events
● Increased risk of noncompliance (may be noncompliant with one medication but adherent with another), age is not a risk factor for adherence
● Increased drug cost
● Do not focus on number of meds but whether the medications are still indicated
● Tailor to individual circumstances, comorbidities, goals of care, preferences, support system, and ability to adhere to medications

How well did you know this?
1
Not at all
2
3
4
5
Perfectly
78
Q

approach to polypharmacy discontinuation

A

Discontinue medications without clear indication (leftovers from acute conditions or transitions of care)

High-risk medications (warfarin, digoxin, hypoglycemic medications)

One drug at a time if condition is stable, more aggressive discontinuation if experiencing side effects that may be due to drugs

Taper down medications at the rate that you would taper them up (opioids, beta blockers, clonidine, gabapentin, antidepressants)

Educate patients on side effects of tapering or discontinuation

Communicate with other providers that may be prescribing

Oral communication is important but not enough → also provide written instructions, especially if more than one person is involved in medications.

Simplifying regimen improves adherence – 80% adherence to one daily dosing versus 50% adherence to 4 times per day dosing

How well did you know this?
1
Not at all
2
3
4
5
Perfectly
79
Q

Discuss the assessment of functional status in the geriatric patient

A

Activities of Daily Living (ADLs)
● Self-care = eating, dressing, bathing, transferring between bed and chair, using toilet, controlling bowel and bladder
● Katz ADL Scale
Instrumental Activities of Daily Living (IADLs)
● To live independently = managing medications, preparing meals, housework, managing finances, using the telephone, driving
● Lawton IADL Scale

How well did you know this?
1
Not at all
2
3
4
5
Perfectly
80
Q

Ms. D and her daughter agree that she has no trouble with her ADLs and is able to perform her IADLs without trouble. She has become more nervous to drive herself at night, and she asks you if she needs to stop driving because of her age.

Discuss the skills necessary for safe driving and medical conditions that can affect the ability to drive.

A

● Memory (dementia)
● Judgment and executive function (dementia)
● Visual and spatial perception (cataracts, glaucoma, poor night vision)
● Physical (arthritis, neurological conditions, hypoglycemia, adverse drug reactions)
● Assessment of driving-related skills battery has not been validated against driving outcomes

How well did you know this?
1
Not at all
2
3
4
5
Perfectly
81
Q

Which patients should be encouraged to limit their driving?

A

Driving cessation can greatly affect quality of life (decreased social integration, increased depression and anxiety, increased risk of nursing home placement) and many patients are very reluctant to give up driving.

Visual and spatial skills are the most relevant predictors of driving impairment.

Dementia and cognitive impairment
● Most patients with early dementia or mild cognitive impairment can pass a driver’s test.
● Diagnosis of dementia should not be the only reason to revoke a driver’s license but experts are unclear about what criteria to revoke in mild dementia.
● By expert consensus, people with moderate dementia should not drive.
● It should be discussed early in the course of dementia that eventually they will have to stop driving.

How well did you know this?
1
Not at all
2
3
4
5
Perfectly
82
Q

Discuss the assessment of patients that may no longer be safe to drive.

A

Driving clinics or driver rehab specialists (occupational therapy) are available but assessments are not standardized.
● Testing takes several hours
● Tests vision, cognition, motor skills
● May be expensive and is usually not covered by insurance

Recommend performance-based evaluation through the Department of Motor Vehicles (DMV) if patient has any of the following:
● If caregiver notices new traffic impairments
● Prominent impairments in key cognitive domains (attention, executive function, visual, and spatial perception)
● Mild dementia

All state DMVs conduct performance-based road tests. If a person fails, license can be revoked.

Mandatory reporting of drivers with dementia in some states but most states have voluntary reporting.

Reporting to the DMV is recommended if clinician feels that they are a danger to themselves or others (for example, if the patient has refused to stop driving or if there is a high risk of crash or injury).

How well did you know this?
1
Not at all
2
3
4
5
Perfectly
83
Q

Ms. D shares that her mother died of Alzheimer disease and did not have an advance care plan or financial will prepared before she lost capacity. Ms. D tells you that she is interested in “getting her affairs in order” and asks you for your recommendation.

What documents do you recommend that this patient consider?

A

● Living will or advance care plan
● Durable power of attorney for health care or medical power of attorney
● Durable power of attorney for finances (DPOAF)
● Last will and testament

How well did you know this?
1
Not at all
2
3
4
5
Perfectly
84
Q

Define financial capacity, how it can affect a patient’s life

A

Financial capacity = ability to independently manage one’s financial affairs in a manner consistent with personal interest
● Loss of financial capacity occurs early in the course of cognitive impairment (one of the first IADLs to decline)
● Can be seen in mild dementia
● Patients with moderate and severe dementia usually cannot manage their finances.
● Can be very psychologically distressing as it is one of the first signs of losing independence
● One of the strongest predictors of caregiver burden
● Makes patients vulnerable for financial abuse

How well did you know this?
1
Not at all
2
3
4
5
Perfectly
85
Q

FNP’s role in assessment of financial capacity

A

● Education about the need for an advance care plan (ACP)
● Durable power of attorney for finances (DPOAF)
● Discuss at the time of diagnosis of dementia
● Recognizing signs of possible impaired financial capacity
● Assessing financial impairment in cognitively impaired adults
● Recommending interventions to help patients maintain financial independence (automatic deposits or withdrawals)
● Knowing when and to whom to make medical and legal referrals
● Legally obligated to report suspected elder abuse including financial abuse
● May be able to refer to neuropsychology, geropsychology, occupational health, or forensic psychiatry depending on what’s available to assess financial capacity.
● If the patient does not have financial capacity, an existing DPOAF can be implemented. If there is no DPOAF then a conservator or guardian is appointed by the state

How well did you know this?
1
Not at all
2
3
4
5
Perfectly
86
Q

Ms. W is a 76-year-old with a history of well-controlled hypertension (HTN) and gastroesophageal reflux disease (GERD) who presents with her daughter. Her daughter is concerned because her mother has seemed more forgetful over the past several months, and she has recently found several unpaid bills in her house. Ms. W has noticed that she sometimes has trouble recalling recent conversations. Last week, she forgot her long-standing hair appointment.

What is the differential diagnosis for memory impairment in an older adult?

A

Dementia
Mild cognitive impairment (MCI)
Depression
Delirium

Others to consider: 
Cerebrovascular dx
hydrocephalus 
hypothyroidism 
Vit B12 deficiency 
CNS infection 
medication effects 
substance abuse
neoplasms
How well did you know this?
1
Not at all
2
3
4
5
Perfectly
87
Q

depression and memory complaints

A

depression should be evaluated in all older patients with memory complaints

More common in women, patients with chronic medical conditions, persistent insomnia, significant stressful events, functional decline, social isolation
● Use the PHQ-9 to assess
● Scale of 0 to 27 (higher scores indicate depression)

How well did you know this?
1
Not at all
2
3
4
5
Perfectly
88
Q

delirium S/S

A
●	Acute decline in attention and cognition
●	Fluctuating course
●	Inattention and disorganized thinking
●	Altered level of consciousness
●	Hallucinations and psychomotor disturbances
●	Altered sleep-wake cycles
●	Emotional lability
●	May be hypo- or hyperactive delirium

Needs urgent evaluation as many causes are life-threatening

Dementia is the leading risk factor for delirium, and two-thirds of delirium occurs in patients with dementia.

How well did you know this?
1
Not at all
2
3
4
5
Perfectly
89
Q

After a thorough discussion with the patient and her daughter, you ascertain that her memory impairment has been gradual and not acute. You review her medication list, which includes hydrochlorothiazide and omeprazole. She has had no changes in her medications, has been taking them as prescribed, and is not taking any other medications. Her thyroid stimulating hormone (TSH) level and vitamin B12 are normal. Although there is some disagreement about whether to obtain imaging, you obtain a brain MRI to rule out cerebrovascular disease and hydrocephalus, and it is normal. PHQ-9 is 4, indicating minimal depression. You believe that she may have dementia.

What is the definition of major neurocognitive disorder (formerly dementia) according to DSM-V?

A
Evidence of decline in one or more cognitive domains
●	Learning and memory
●	Language
●	Executive function
●	Complex attention
●	Perceptual-motor
●	Social cognition

The cognitive deficits interfere with independence in daily activities, and at minimum, needs help with complex activities of daily living (such as paying bills)

The cognitive deficits are not exclusively due to delirium

The cognitive deficits are not explained by another mental disorder (such as schizophrenia or major depressive disorder)

90
Q

Describe the different etiologies of dementia, and how the clinical presentation may differ among them.

A

Important to identify (if possible) because can change treatment and will give more prognostic information to the patient and family

Alzheimer disease (50%-80% prevalence)
●	Rapid forgetfulness, especially of new material

Frontotemporal dementia (12%-25%)
● More trouble planning tasks
● More behavioral disturbances and disinhibition

Mixed etiology (10%-30%)

Vascular dementia (10%-20%)
● More quick onset, more language problems
● Less forgetful than AD especially if cues available

Lewy body dementia (5%-10%)
● May have more psychiatric symptoms (eg, psychosis) early in disease

91
Q

Describe the risk factors and the protective factors associated with the development of dementia

A
Risk factors
●	Increased age
●	Stroke
●	Hypertension
●	Apolipoprotein E status
●	Family history for AD7

Protective factors
● Higher levels of education
● Physical activity
● Moderate alcohol intake

92
Q

How is dementia diagnosed?

A

Reliable information is obtained from the informant as well as cognitive behavioral testing.

More likely to come to attention when symptoms involve behavioral problems or dependence on activities of daily living (ADLs).

Subjective memory complaints may be less likely to yield diagnosis of dementia than objective memory complaints, although both should be evaluated by validated tests.5

Subjective complaints may be more associated with depression, lower levels of education, older age, male sex, and poorer overall health.5

93
Q

Tests available to diagnose dementia

A

Mini mental status exam (MMSE)

Memory impairment screen

Clock drawing test

General Practitioner Assessment of Cognition

More comprehensive tests:

  • Modified MMSE
  • Hopkins Verbal Learning Test or Word List Acquisition Test
  • Cognitive Assessment Screening Test
  • Psychogeriatric Assessment Scales
  • Clinical dementia rating scale
94
Q

MMSE

A

Most widely studied cognitive test

Domains: orientation, registration, attention and calculation, recall, language

Should be adjusted for patient’s age, may be influenced by education level (norms have been published but no score conversions are available)

Most helpful for moderate dementia, less for mild dementia

Free of charge if used from practitioner’s memory, original paper, or source authorized by publisher
● Takes 7 to 10 minutes to administer
● Positive test usually is a score less than 23 to 24 out of 30
● Sensitivity = 71% to 92%
● Specificity = 56% to 96%

95
Q

Memory Impairment Screen

A

● Tests recall ability
● Patient given nouns in 4 groups (animal, city, vegetable, musical instrument)
● Then given a diversionary task
● Ask to recall nouns: 2 points each if no prompting needed, 1 point for prompted answer
● Allows use for research and license available for commercial use
● Takes 4 minutes to administer
● Cut-off score of 4 yields sensitivity of 80% and specificity of 96%

96
Q

Clock Drawing Test

A

● Tell patient to “put numbers on the face of a clock” and “make the clock say 10 minutes to 11”
● Can distinguish between normal aging and mild dementia
● Several different ways to score it with differing reliability
● May increase the sensitivity of MMSE to pick up very mild dementia

97
Q

You administer the MMSE, and Ms. W scores 24/30. She has a college education and because you are uncertain of whether she has mild cognitive impairment or mild dementia, you send her for formal neuropsychiatric testing. Because you did not have access to more detailed testing in your clinic, you sent her for neuropsychiatric testing. She was diagnosed with mild dementia, and the patient and family would like to discuss treatment options.

Discuss the classes of medication currently available for the treatment of dementia.

A

Cholinesterase inhibitors (donepezil, galantamine, rivastigmine)

  • FDA approved for mild to moderate AD, NOT severe AD
  • Side effects: N/V/D, dizziness, weight loss
  • contraindications: cardiac conduction abnorms, gastric ulcers, hx bleeding

N-Methyl-D-aspartate (NMDA) receptor antagonist (memantine)

  • FDA approved for mod to severe dementia
  • not beneficial in mild-mod dementia as monotherapy, add to cholinesterase inhibitor as dementia progresses
  • side effects: HA, sedation, constipation, agitation
98
Q

medications that have no or inconsistent effect for tx of dementia

A
o	Anti-inflammatory
o	Estrogen
o	Statins
o	Antioxidant vitamins
o	Ginkgo biloba
o	Acetyl-L-carnitine
o	Lecithin
99
Q

Which medications would you recommend for this patient with a diagnosis of mild dementia, most likely due to AD? How would you counsel this patient and family on what to expect from these medications?

A

A cholinesterase-inhibitor
● May show symptomatic improvement or slowing of progression for 6 to 12 months, with possible benefits at 2 to 3 years
● Studies have not shown a significant difference between donepezil and placebo in terms of caregiver well-being, time of caregiving, or costs of care

100
Q

If the had been diagnosed with mild cognitive impairment, would this change your recommendations for her treatment?

A

● No FDA-approved treatment

● Cholinesterase inhibitors have not been shown to slow progression to AD, although the expert consensus guidelines say that it may be considered for patients with MCI.

101
Q

If the patient been diagnosed with vascular dementia, would this change your recommendations for her treatment?

A

More focus on aggressive treatment of hypertension and diabetes
● Consider aspirin and lipid-lowering agent5
● Consider cholinesterase-inhibitor in mixed vascular and Alzheimer disease

102
Q

What nonpharmacologic interventions have been studied in patients with mild dementia?

A

Cognitive training
o Targeted practice and training in specific domains of cognition, usually memory and executive functioning
o Has not shown significant difference in mild dementia compared to control

Cognitive rehab programs
o More geared toward improving functionality in everyday life
o May improve cognitive function in patients with MCI
o Has not shown significant difference in mild dementia compared to control

Reminiscence therapy
o Allows patient to revisit memories and experiences
o Significant improvement in cognitive and depressive symptoms

103
Q

Ms. W’s daughter is concerned about the behavioral disturbances that can be associated with progressive dementia. She is wondering what your approach to those symptoms will be.
Discuss the behavioral disturbances that are associated with progressive dementia and their treatment recommendations

A

There are no psychotropic agents that are FDA approved for treatment in dementia, although there are several medications that are commonly used

Agitation with delirium or psychosis
o Atypical antipsychotics (risperidone, olanzapine, quetiapine) are commonly used
o May increase risk of metabolic syndrome and stroke
o Black box warning for atypical antipsychotics issued by FDA in 2005 for elderly patients with dementia-related psychosis as they found an increased rate of death compared to placebo.
o If these medications are used, they should be targeted at a specific symptom with plans to taper and remove medication within 6 months.

Agitation and aggression
o	Anticonvulsants (divalproex, carbamazepine)

Depression or anxiety
o Selective serotonin reuptake inhibitors (SSRIs) (such as sertraline and citalopram) or buspirone
o Avoid tricyclic antidepressants (TCAs) as they may worsen agitation and increase risk of falls.
o Benzodiazepines may be used for short-term treatment of anxiety5 although this may worsen confusion, increase risk of falls, and cause paradoxical agitation
o No consensus on choice of medication

Insomnia - consider trazodone

All of these symptoms can occur at any time throughout the course of dementia, although they are more common as dementia advances.

Nonpharmacologic interventions to calm and reorient patients include music, videotapes of family members, maintaining a familiar environment, and caregiver training.

104
Q

Ms. W and her daughter ask you if there are other arrangements they should put in place at this time.

What recommendations do you have for this family regarding legal arrangements that should be made at this time?

A

Persons with AD may live 3 to 20 years (average 8 years)

Early planning allows patient and family to make financial, legal, and medical decisions while patient still has capacity to make decisions for themselves.

The financial and health care surrogates would only be asked to make decisions if the patient were to lack the capacity to make decisions for themselves.

Medical
o Designation of health care power of attorney
o Discussions of quality of life, boundaries to medical interventions, and advance care planning
o Creating a living will

Financial
o Designation of financial power of attorney
o Adding financial power of attorney (POA) to financial accounts
o Creating a (financial) will

105
Q

Discuss how living arrangements will change and the safety issues that will arise with progressive dementia.

A

Living arrangements: Eventually patients will be unable to live alone. Patient and family should consider whether they have the time and financial resources to care for a patient at home, or whether to consider nursing home placement.

Safety:
o	Monitor medications early on
o	Limit or stop driving (many states do evaluations for free through the Department of Motor Vehicles)
o	Monitor use of cooking and appliances
o	Remove firearms from the home
o	Home safety assessments

Caregiver self-care is very important as the emotional and financial stresses of caring for a family member with dementia can be significant

106
Q

A 57-year-old woman presents to primary care practice for follow-up. She has a history of well-controlled hypertension and chronic anxiety but has otherwise been healthy. While on-call for the practice, you receive a message from her husband that she has been acting strangely today. On the phone, he tells you that she has had a few days of nausea, vomiting, and diarrhea after a recent visit from their grandkids. She has been able to keep down some fluids, and she thought she was slowly getting better this morning. She has been trying to take some anti-nausea medicine, but none of her pills have stayed down since the onset of her illness 3 days ago. She has been tired and irritable since this started, but he is more concerned tonight because she is “talking out of her head and acting a little crazy” and has never behaved like this in the past.

How is this alteration in her behavior best categorized? What features lead you to this definition?

A

According to the Diagnostic and Statistical Manual of Mental Disorders, 5th edition (DSM-V), delirium is characterized by 4 key features:

● Disturbance in attention (decreased ability to direct, focus, and sustain attention) and orientation to the environment
● Development of the disturbance over a short period of time (usually hours to days) which fluctuates over the course of the day
● A change in cognition or the development of perceptual disturbance that is not explained by a preexisting condition (eg, dementia)
● The disturbance is not caused by a severely reduced level of arousal, such as coma

Given the sudden onset of her condition in the setting of no prior history, this situation would be best classified as an acute delirium.

107
Q

A 57-year-old woman presents to primary care practice for follow-up. She has a history of well-controlled hypertension and chronic anxiety but has otherwise been healthy. While on-call for the practice, you receive a message from her husband that she has been acting strangely today. On the phone, he tells you that she has had a few days of nausea, vomiting, and diarrhea after a recent visit from their grandkids. She has been able to keep down some fluids, and she thought she was slowly getting better this morning. She has been trying to take some anti-nausea medicine, but none of her pills have stayed down since the onset of her illness 3 days ago. She has been tired and irritable since this started, but he is more concerned tonight because she is “talking out of her head and acting a little crazy” and has never behaved like this in the past.

What historical information is most important in determining a cause of her symptoms?

A

Recognition of the diagnosis is the most important aspect to appropriate management. Some estimates state that up to 70% of delirium goes unrecognized in the inpatient and outpatient settings.

Because the patient will most likely be unable to provide an adequate history, details should be elicited from family or witnesses nearby. Key points of history that should be investigated are signs or symptoms of recent infection, history of organ failure, a medication list, history of substance use, and psychiatric history including recent evidence of depression.

108
Q

A 57-year-old woman presents to primary care practice for follow-up. She has a history of well-controlled hypertension and chronic anxiety but has otherwise been healthy. While on-call for the practice, you receive a message from her husband that she has been acting strangely today. On the phone, he tells you that she has had a few days of nausea, vomiting, and diarrhea after a recent visit from their grandkids. She has been able to keep down some fluids, and she thought she was slowly getting better this morning. She has been trying to take some anti-nausea medicine, but none of her pills have stayed down since the onset of her illness 3 days ago. She has been tired and irritable since this started, but he is more concerned tonight because she is “talking out of her head and acting a little crazy” and has never behaved like this in the past.

What conditions should be considered within your differential?

A

The differential diagnosis for delirium should include a wide range of processes such as electrolyte or metabolic imbalances, infections, intoxications, withdrawals, psychiatric illness or onset of dementia, or vital sign instability such as hypoxia or hypotension.

For more severe episodes of delirium, seizure or stroke should also be included on the list of possible causes. One of the greatest risk factors for the development of delirium is age, as the elderly are most susceptible to risk factors such as change in surroundings, depression, medication changes, or acute stressors such as surgery or hospitalization due to illness.

For all patients, up to 30% of delirium cases may be due to medications or alterations in their metabolism.

For this case, inquiring about her past medical history reveals a clue. Review of Mrs. Hayes’s history would reveal that she had been taking alprazolam 3 times a day for her anxiety for quite some time, but she has been unable to tolerate her medications over the past several days. This sudden withdrawal of a benzodiazepine is the most likely source of her symptoms.

109
Q

Review of Mrs. Hayes’s history would reveal that she had been taking alprazolam 3 times a day for her anxiety for quite some time, but she has been unable to tolerate her medications over the past several days. This sudden withdrawal of a benzodiazepine is the most likely source of her symptoms.

What is the most important step in addressing this situation?

A

Most practitioners consider any episode of delirium to be a medical emergency that requires rapid evaluation and intervention. Because a full history and physical are not possible over the phone, it is important that this patient is brought to the emergency department for a full evaluation right away.

110
Q

You see Mrs. Hayes over the next several years and she has no recurrence of her symptoms. She develops some arthritis and mild renal insufficiency over the years, but otherwise does well. As she and her husband age, they eventually move into a senior community in town, and her daughter begins to bring her in for her routine visits. Now age 73, her daughter reports that this past month she was called by the senior community administration because her mother has not been paying her bills. By her own history, she admits to having some “memory problems” but still feels she is able to take care of most of her daily affairs.

As you see her in your office, does she need an urgent delirium workup for her recent symptoms?

A

In this case, a full history is vital in establishing need for a further workup. Most likely, Mrs. Hayes does not have acute delirium in this case, based on the slow onset of the concerning symptoms and her ability to focus on questions posed to her. There is no history of difficulty with more basic activities of daily living (ADLs), and this is more likely an early presentation of dementia rather than an acute delirium. Differentiating between delirium and dementia can be difficult, especially in later stages of dementia, and is made more difficult in that dementia is a significant risk factor for the development of delirium.

Assessment tools such as the Confusion Assessment Method (CAM) for delirium and the Mini Mental Status Exam (MMSE) for dementia may help confirm each diagnosis. A history of sudden onset or fluctuation during the day is a key indicator that there is delirium with or without underlying dementia.

It is reasonable to screen for additional psychiatric features in this situation. Other psychiatric conditions, such as depression, often manifest as an acute delirium or dementia-like picture in older patients. In younger patients other psychiatric illnesses, like schizophrenia or manic syndromes, may be initially presented as delirium.

111
Q

After the passing of her husband a few years later, Mrs. Hayes eventually requires 24-hour care and is moved to a nursing home near her daughter because of her worsening disease. Her daughter brings her in for an urgent visit late one day because her mother has been more withdrawn and confused over the preceding 24 hours.

What common triggers must be evaluated for in this presentation of acute delirium?

A

As mentioned earlier, the elderly are at the greatest risk of developing delirium due to an often reduced capacity to handle physical and psychological stressors.

Elderly patients often respond to a multitude of conditions by developing delirium, often evidenced by withdrawal, confusion, or agitation. The most common of these conditions include infections (such as urinary tract infections, pneumonia, or skin infection), pain, sleep deprivation (leading to what is commonly called “sundowning”), depression, volume depletion, or medication effects.

Delirium is also a common feature seen in patients with advanced malignancy or who are recently postoperative from complex procedures.

112
Q

In addition to treating the precipitating cause of delirium such as an acute infection or adjusting suspected medications, what other interventions can be helpful for the prevention and treatment of delirium?

A

Primary interventions that are effective during treatment and for prevention of delirium should focus on environmental factors, such as reorientation, appropriate sleep hygiene, visual or hearing aids if these are impaired, maximizing mobilization, and avoiding physical restraints unless the patient poses a significant risk to themselves or others.

One component of the environment that may be vital to a delirious patient (and that usually escapes the attention of providers) is interpersonal need of social interaction, which can continue to exist despite a loss of communication or comprehension. In many situations, the mere presence of a family member or even medical staff can be enough to alleviate the stress of isolation often seen in delirious and demented patients.

Of course in many cases, pharmacotherapy is an important component in the effective treatment of acute delirium. This becomes more important especially for patients with hyperactive or aggressive delirium. Pharmacotherapy focuses on altering common neurotransmitters involved in delirium such as acetylcholine, dopamine, serotonin, and gamma-aminobutyric acid (GABA).

113
Q

What is the most common cause of hearing loss in people over 80 years old?

A. Otosclerosis
B. Cerumen impaction
C. Presbycusis
D. Otitis media

A

C. Presbycusis

114
Q

Which of the following is not a risk factor for falling in community-living older adults?

A. Body mass index (BMI) of 25 kg/m2
B. Previous falls
C. Gait instability
D. Vision impairment

A

A. Body mass index (BMI) of 25 kg/m2

115
Q

Which of the following medications should be used as first-line therapy in the treatment of depression in patients older than 60?

A. Sertraline
B. Duloxetine
C. Nortriptyline
D. Mirtazapine

A

A. Sertraline

116
Q

Which of the following statements is true about financial capacity in elderly patients?

A. Financial abuse is a form of elder abuse.
B. Losing the ability to make financial decisions is an expected part of aging.
C. Losing the ability to make financial decisions occurs late in the course of cognitive impairment.
D. A durable power of attorney for health care document can be used by a patient’s health care agent to make financial decisions.

A

A. Financial abuse is a form of elder abuse.

117
Q

Which of the following statements is true about driving in older adults with cognitive impairment?

A. Adults with severe dementia can drive safely during daylight hours.
B. The mini mental status exam (MMSE) can be used to infer driving capacity.
C. Tests of visuospatial skills are the most relevant predictors of driving impairment.
D. Cessation of driving does not affect quality of life in older adults.

A

C. Tests of visuospatial skills are the most relevant predictors of driving impairment.

118
Q

Which of the following is not a risk factor for the development of dementia?

A. Cardiovascular disease
B. Family history of dementia
C. Increased age
D. Moderate alcohol intake

A

D. Moderate alcohol intake

119
Q

An 85-year-old man with a history of well-controlled hypertension presents to primary care clinic with his daughter to discuss memory loss. They have both noticed that he is increasingly forgetful. He has forgotten some doctors’ appointments and forgot to pay his electric bill. He is able to perform all of his activities of daily living (ADLs), and has no trouble with swallowing or language. What is the most likely diagnosis?

A. Normal aging
B. Mild cognitive impairment
C. Alzheimer disease
D. Vascular dementia

A

B. Mild cognitive impairment

120
Q

What is the most common type of dementia?

A. Alzheimer disease
B. Vascular dementia
C. Lewy body dementia
D. Frontotemporal dementia

A

A. Alzheimer disease

121
Q

What is the most common complication in advanced dementia?

A. Feeding difficulties
B. Pneumonia
C. Hip fracture
D. Urinary tract infections

A

A. Feeding difficulties

122
Q

Which of the following patients with dementia is eligible for hospice care under current Medicare guidelines?

A. Cannot dress without assistance, is continent of bowel and bladder, and has decreased oral intake without weight loss
B. Has fewer than 6 intelligible words per day, can ambulate with assistance, history of cystitis
C. Can hold head up independently but cannot sit up independently, two hospitalizations for aspiration pneumonia
D. All of the above

A

C. Can hold head up independently but cannot sit up independently, two hospitalizations for aspiration pneumonia

Hospice is a medical service that cares for patients with end-stage illness when the primary focus is on symptom management at the end of life. In the United States, Medicare sets hospice requirements; most other insurance plans have adopted these requirements. Patients qualify for the Medicare hospice benefit if two clinicians (usually the referring clinician and the hospice medical director) agree that the patient likely has less than 6 months to live if the illness were to take its natural course.
While it is difficult to predict a patient’s medical trajectory with certainty, current hospice eligibility guidelines identify patients with dementia as having less than a 6-month prognosis if they have a Functional Assessment Staging tool (FAST) stage 7c (unable to ambulate independently) and at least one of the following medical conditions: aspiration pneumonia, pyelonephritis or upper urinary tract infection, septicemia, pressure ulcer (multiple, stage 3 or 4), recurrent fever after treatment with antibiotics, and eating problems (such as decreased intake, weight loss, or albumin level less than 2.5 g/dL).

123
Q

What is the overall median survival time after diagnosis of dementia?

A. 6 - 12 months
B. 2- 3 years
C. 4 -5 years
D. 8 - 10 years

A

C. 4 -5 years

The median survival time after diagnosis of dementia is 4.5 years when all patients with dementia are considered but changes based on age of diagnosis. For people diagnosed in their 60s, the median survival time after diagnosis is approximately 11 years. For people diagnosed in their 90s, the median survival time is approximately 4 years.

124
Q

Which of the following medications has been shown to slow cognitive deterioration in patients with moderate to severe Alzheimer disease (AD) but not in those patients with mild AD?

A. Donepezil
B. Rivastigmine
C. Memantine
D. Galantamine

A

C. Memantine

Memantine, an N-methyl-d-aspartate (NMDA) receptor antagonist, has been shown to slow cognitive deterioration with moderate to severe AD, but has not been shown to benefit patients with mild AD. Cholinesterase inhibitors such as donepezil, rivastigmine, and galantamine have been shown to decrease cognitive decline in patients with mild to moderate AD.

125
Q

A 67-year-old woman presents to primary care clinic to establish care. She is healthy, and takes no prescription medicines. Other than some osteoarthritis for which she takes 500 mg of acetaminophen a few times per week, she states she has no medical problems. She has not seen a doctor since she was able to obtain her Medicare two months ago. She asks for a Medicare routine health maintenance annual physical.

What services are required during the Welcome to Medicare visit?

A

Medicare does not pay for a classic “annual physical.”
● They pay for a one-time screening physical exam for the purpose of health promotion and disease prevention.
● This is called the IPPE, or Initial Preventive Physical Examination
● It is a screening visit, and not meant to be an evaluation for all her medical problems.
● This is a one-time Medicare benefit that patients can receive.
● After the initial 12 months of eligibility have passed, or after they has received the one-time IPPE, they can receive an Annual Wellness Visit (AWV) (which does NOT include an annual physical) yearly

The one-time IPPE must include all of the following elements:

1) History
a. Medical history
▪ Medicines, supplements
▪ Alcohol, tobacco, drug use
▪ Diet and exercise
b. Risk Factors for depression – use a screening tool (eg, Beck depression inventory, Patient Health Questionnaire 2 [PHQ-2])
c.Functional status and safety assessment
▪ Use any standardized screening tool to assess (at a minimum):
▪ Hearing
▪ Activities of daily living (ADLs)
▪ Fall risK
▪ Home safety

2) Examination
a. Measurements: height, weight, BMI, blood pressure
b. Vision
c. Other screening exam based on history and risk

3) End-of-life planning (verbal or written information to be given)
a. Advance directive planning

4) Counselling
Based on the information obtained above, offer education, counselling, referral as appropriate
a.Provide patient with a written plan (a checklist) for the following screening
▪ EKG as appropriate
▪ Medicare-covered screening tests (essentially, US Preventive Services Task Force A and B recommendations)

126
Q

What CPT and ICD-10 codes do you use for the Welcome to Medicare visit?

A

● CPT code is G0402 (for the IPPE)
● ICD-10 code is Z00.0 series (Encounter for general adult medical examination)

In truth, CMS does not require a diagnosis code for this covered service and they state “you may use any diagnosis code you choose” as it will be covered by Medicare from the CPT alone.
(If you do a screening electrocardiogram (ECG), the CPT codes change slightly to G0403-405 depending on whether an electrocardiogram, the interpretation, or both are provided.)

127
Q

What if you address her osteoarthritis, order x-rays, review her x-rays, prescribe a new medicine, and refer her to physical therapy, in addition to the Welcome to Medicare visit? How does that change your coding?

A

You have provided an additional E/M service in addition to her screening physical. You can add a CPT code 99213 for a mid-level E/M visit, with a 25 modifier since you are adding an additional service, with an ICD-10 of M19.90 for generalized osteoarthritis.

128
Q

How does billing change if you are a resident physician?

A

You need to add a GE modifier if you see the patient on your own or a GC modifier if you see the patient with your attending physician.

129
Q

The patient returns after one year and shares that her osteoarthritis is much better with the new NSAID and physical therapy you recommended. She wants to come to see you for her annual physical this year, but she feels great with no new issues and only wants to do what Medicare will pay for.

What services will Medicare cover for this patient this year? How is that different than last year?

A

Medicare does not pay for an annual physical. After the one-time IPPE, they do not pay for even a screening physical. For all subsequent years, the Annual Wellness Visit (AWV) is intended to identify problems, to prevent problems, and to make sure the patient receives the appropriate screening, education, guidance, and referral. It is not a “classic annual physical.” It is important to provide the patient with the information about what these visits cover so that they have the appropriate expectations for these visits.

For the Annual Wellness Visit (AWV), these are the required elements:
1) History
o Health Risk Assessment (HRA)
▪ This assesses: Demographic data, self-assessed health status, psychosocial risk, behavioral risk, ADLs (self-care–bathing/dressing; life skills–shopping, meals, banking)
o Standard Patient History
▪ Standard – past medical, family, and social history
▪ Depression Screening (use a tool, eg, Beck depression inventory, PHQ-2)
▪ Assessment functional ability (direct observation or a screening tool)
▪ Activities of daily living (ADLs)
▪ Fall risk
▪ Hearing assessment
▪ Home safety

2) Assessment
o Measurements: height, weight, BMI, blood pressure
▪ Other measurements based on identified risk (waist circumference)
o List other current providers
o Cognitive impairment
▪ By direct observation, care provider reports, or by screening tool

3) Counselling
o Establish a written screening schedule for the next 5 to 10 years (including):
▪ USPSTF A and B screening recommendations by age
▪ Advisory Committee on Immunization Practices (ACIP) vaccines recommendations
▪ Patient’s prior screening history
▪ This is the PPPS (Personalized Prevention Plan of Service)
o Establish a list of risk factors and conditions of patient
▪ Medical diagnoses
▪ Mental health conditions
▪ List of treatment options, with risks and benefits
o Personalized health advice–counselling or referral to programs for:
▪ Risk reduction, wellness
▪ Fall prevention
▪ Nutrition
▪ Physical activity
▪ Tobacco cessation
▪ Weight loss

130
Q

What CPT and ICD-10 codes apply to the AWV?

A

● CPT code: G0438 for the initial AWV, and all subsequent AWVs are G0439
● ICD-10: Z00.0 series (Encounter for general adult medical examination)

Note: You may choose to do a problem-based visit for Medicare patients since most older patients have medical problems and work in preventive services to these visits. However, these Medicare wellness visits provide a strategy and structure to make sure you preserve time for these complex patients to get the screening and counselling done, ensure it is comprehensive, and also get paid for it.

131
Q

types of stroke

A

Ischemic:

  • thrombosis - blockage of blood flow d/t arteriosclerosis
  • embolic - blood clot formed elsewhere travels to cerebral circulation

Hemorrhagic/intracerebral hemorrhage:

  • primary: majority of ICH r/t chronic HTN
  • secondary: ICH d/t cerebrovascular abnorm (tumor, impaired coag)
  • subarachnoid hemorrhage: typically d/t aneurysm rupture, AVM, or bleeding d/o
132
Q

stroke RF

A

modifiable:
HTN, DM, smoking, HLD, obesity, poor diet/nutrition, physical inactivity, afib, ETOH
HTN is most important independent RF

unmodifiable: older age, men, AA, fam hx

133
Q

stroke S/S

A

sudden numbness/weakness of face, arm, leg, esp one side
sudden confusion or trouble speaking or understanding speech
sudden trouble seeing in one or both eyes
sudden trouble walking, dizziness, loss of balance or coordination
sudden severe HA with no known cause

134
Q

TIA presentation

A

transient neuro symptoms that resolve
lasts few mins to hours
resembles early stroke - numbness, weakness, flaccidity, visual changes, ataxia, dysarthria

precedes 10% of strokes

typically requires 24 hr observation ER
lifestyle mod to prevent ischemic stroke
use ABCD score to determine course of tx

135
Q

ischemic stroke course/symptoms

A

ischemic event evolves over course of a few hours
if carotid involved: ipsilateral eye affected = amaurosis fugax: transient painless loss of vision

hemispheric ischemia: numbness/weakness of contralateral face/limbs

language diff
cognitive/behavior changes

136
Q

vertebrobasilar infarct

A
back of brain
vertigo
N/V
nystagmus
diplopia 
dysconjugate gaze
137
Q

ABCD score for TIA

A
Age >/= 60 = 1 pt
BP >/= 140/90 = 1 pt 
Clinical presentation:
- unilateral weakness w/ or w/o speech changes = 2 pts
- speech changes, no unilateral weakness = 1 pt 
Duration:
- >/= 60 min = 2 pt 
- = 59 min = 1 pt 
Diabetes = 1 pt 

1-3 points = 1% 48hr stroke risk
- neuro consult, possible outpt tx and eval

4-5 points = 4.1% 48 hr stroke risk
- hospitalization justified

6-7 points = 8.1% of 48 hr stroke risk
- hospitalization beneficial

138
Q

primary ICH presentation

A

typically no prodrome
sudden onset of hypertensive crisis - 180/120
almost always occurs while awake w/activity
sudden onset HA, unilateral facial droop, slurred speech, eye deviation
5- 30 min progression
paralysis, aphasia

139
Q

RED FLAGS of ICH

A
vomiting
SBP > 220 
severe HA 
coma
decreased LOC 
s/s progression over mins- hrs
140
Q

SAH presentation

A
abrupt onset
"worst HA of life" thunderclap HA 
N/V dizziness
LOC
meningeal irritation 
atypical HA for weeks leading up to event
seizures
141
Q

stroke diagnostics

A

NIH stroke scale on ALL pts w/ possible stroke
MUST fo dx studies to determine ischemic vs hemorrhagic:
- CT (most common & time sensitive) 1st test to complete
- ischemic areas will not be visible immediately after s/s onset
- very sensitive for hemorrhage
- MRI more sensitive in acute phase of ischemic stroke

if possible seizure: ECG, CT, ABG/SPO2, EEG

CBC, PT, PTT, serum glucose, Cr
- Do NOT do before CT, dont delay

Carotid US
TEE: obtain clear images of heart and blood clots (afib?)
TTE: finds source of clots in heart that may have traveled to brain to cause stroke
cerebral angiogram
holter monitor

142
Q

Goal of stroke workup/tx

A

obtain CT & administer thrombolytics (if indicated) within 45- 60 mins of ED arrival

143
Q

NIH stroke scale score/ DC needs

A

rehab needs:
>/=14 severe = long term care
6- 13 adequate: acute inpt rehab
= 5 mild: 80% DC home

144
Q

stroke management

A

immediate transport to stroke facility

145
Q

PCP role in stroke

A

primary and secondary interventions:
BP control, anti-thrombotic therapy, smoking cessation, diet/nutrition, physical activity, OSA mgmt, BG control, HLD mgmt

educate on stroke s/s & instruct to immediately call 911

146
Q

Patient presents to PCP office days after probable TIA s/s reported, but has no current symptoms of neuro deficits. How do you manage?

A

Can be evalated/ tx outpatient

evaluate probable cause: afib, carotid stenosis
- get carotid US & ECG

RF management

147
Q

ischemic stroke inpt mgmt

A

neuro consult
tPA & revacularization (thrombectomy)
tPA given within 4.5 hrs of s/s onset

148
Q

hemorrhagic stroke inpt mgmt

A
BP mgmt 
reversal of anticoag if on one
- Warfarin = Vit K 
- ASA or Plavix = give platelets 
- heparin or lovenox - protamine sulfate
149
Q

tPA inclusion criteria

A

> 18 y.o.
clinical dx of ischemic stroke
s/s onset < 180 min before tPA admin

150
Q

tPA exclusion criteria

A

PO anticoag
DM & previous stroke hx
> 80 y.o.
NIHSS > 25

151
Q

antiplatelet tx for stroke

A

prevention
beneficial regardless of age, gender, BP, or DM dx

ASA: standard preventative care

Warfarin/other DOACs: pt @ risk for embolism/hypercoaguable state
- Afib, LV dysfunction + CHF, artificial heart valves

thienopyridines (Plavix or Ticlid): allergic to ASA

152
Q

dementia

A

progressive loss of memory & behavioral changes, personality changes, language disturbances which interfere with independence in ADLs

Alzheimer’s dx
vascular dementia
Lewy body dementia

153
Q

dementia RF

A

mild cognitive impairment
HTN, DM, HLD, peripheral vascular occlusion
fam hx, old age, Down syndrome

154
Q

lewy body dementia S/S

A
visual hallucinations
motor impairments 
postural instability 
sleep disturbances 
can have psychosis

more psychiatric symptoms

155
Q

vascular dementia S/S

A

quicker onset
more language problems
less forgetful than AD

156
Q

Alzhiemer dx S/S

A

rapid forgetfulness, esp of new material

157
Q

dementia presentation

A

memory loss, personality changes, language disturbances, diff w/ ADLs

158
Q

dementia diagnosis based on

A

no single standardized test
dx of exclusion

  • detailed hx from fam or caregivers
  • complete physical and neuro exam
  • brief mental status or cognitive screening tests can be completed to ID pts most likely to benefit from more comprehensive assessment
159
Q

dementia labs/imaging

A

CBC, CMP. TSH, B12, folate, serum drug levels

baseline brain imaging w/ non-contrast CT or MRI

160
Q

dementia mgmt

A

depends on stage of dementia

  • tx all correctable factors that may impair cognition to improve ADLs & delay disability
  • activities that promote & enhance cognition & social engagement
  • durable POA
  • address safety concerns - driving, kitchen
161
Q

Alzheimer’s diagnosis

A
no single dx study 
CBC, BMP, thyroid, B12, folate, serum drug levels 
possible imaging 
brain biopsy postmortem = definitive dx
initial cognitive testing (clock)

REFER to neuro to confirm

162
Q

AD management

A

meds delay progression, not curative

1st line = cholinesterase inhibitors

  • refer to neuro for med mgmt
  • Donepezil - mild- mod AD

NMDA (memantine) - mod- severe AD w/ cholinesterase inhibitor

SSRIs - Citalopram for neuropsych s/s like agitation

CBT - mold to mod AD

163
Q

early-stage dementia

A
memory loss, poor judgement 
withdrawn, depressed 
perceptual disturbances
personality changes 
time & spatial disorientation
164
Q

middle-stage dementia

A
recent & remote memory worsens 
increased aphasia (slowed speech & understanding) 
apraxia, hyperorality
disorientation to place & time 
restlessness or pacing 
perseveration, irritability
loss of impulse control
165
Q

late-stage dementia

A
incontinent bowel/bladder
loss of motor skills, rigidity 
decreased appetite, dysphagia 
agnosia, apraxia 
severely impaired communication 
possible inability to recognize family members or self in mirror 
loss of most or all self-care abilities 
severely impaired cognition, depressed immune system
166
Q

dementia indications for referral

A

neuro consult helpful w/ unclear clinical pic
unusual presentation or depression present - consult
endstage dementia - hospice referral
speech therapy for swallowing or dysphagia in late stages
driving, kitchen and home safety evals by OT

167
Q

criteria for hospice in dementia

A

bed bound status

late stage of dementia

168
Q

delirium

A

acute change in MS

often only indicator in older adults of underlying physical illness: infection, MI, drug toxicity/withdrawal

169
Q

delirium symptoms

A

hallmark –> clouding of consciousness w/ inability to focus, sustain, or shift attention
change in cognition, including impairment in short-term memory, disorientation, & perceptual disturbances
symptoms develop rapidly and fluctuate in severity

170
Q

delirium & dementia

A

delirium superimposed on dementia

pt w/ underlying dementia @ greater risk for delirium w/ acute illness

171
Q

delirium tx

A

ID & tx of precipitating causes

management of symptoms such as agitation, restlessness, and hallucinations

172
Q

pseudodementia

A

depression in older adults can lead to memory loss, attention deficits & problems with initiation

cognitive symptoms often persist as residual symptoms in patients for whom depression has remitted

cognitive problems that coincide with onset of MDD ep in context of previously normal cognition - more likely d/t depression

significant depression related cognitive impairment can result in dementia

follow up with older adults w/ depression related cognition impairment even after depression remits

173
Q

dementia screening

A

Not recommended by USPSTF

is a part of the Medicare Annual Wellness: MMSE, Mini-Cog

174
Q

immunizations recc by CDC and Medicare for older adults

A

pneumococcal
influenza
herpes zoster
tetanus or TD

175
Q

screenings for older adults

A
dementia 
hearing 
vision
glaucoma 
falls
176
Q

Advanced directives

A

role of PCP to review with pt

allows for autonomy regarding manner & location of death, relieves family burden and conflict

177
Q

living will

A

describes patients wishes about resuscitation, hospitalization, tx goals & limits, health care proxy

178
Q

polypharmacy

A

use or misuse of 5 or more drugs not clinically indicated & interact w/ each other
drug distribution & clearance affected by normal aging

can lead to MS change, sedation, falls

Mgmt
review all meds @ each visit, compare w/ BEERS list, encourage updated med list
start low & go slow w/ prescribing

179
Q

dehydration

A

prevalent in older adults
sodium electrolyte imbalance most significant (confusion, weight loss, FTT)

causes: environmental, increased metabolic demands, pharmacologic factors, normal aging changes

presents: confusion, lethargy, rapid weight loss, functional decline
- constipation (lack of water intake)
- orthostatic BP, rise in pulse, dry MM, poor skin turgor
- BUN/Cr ration 25:1
- sodium > 148 (acute mental status changes)
- elevated hematocrit

180
Q

fluid deficit determined by

A

pre-illness weight - current weight = fluid deficit

rehydrate oral (preferred)
clysis (non emergent subcutaneous upper arm/unable to take PO)
IV hydration - fastest
- 1/2 of calculated fluid loss in 1st 24 hrs, min 1500mL

181
Q

fluid education in older adults

A

6- 8 ox water/juice a day
take full glass with meds

caffeine, alcohol (beer) diuretics & should be avoided

182
Q

Falls assessment

A

complete fall assessment DDROPP - diseases, drugs, recovery, onset, prodrome, precipitants

Romberg test w/ sternal nudge & check for nystagmus, CV and neuro exam, postural VS, mobility, strength, cognition, vision & hearing

CBC, lytes, BUN, Cr, glucose, OB, ECG

183
Q

failure to thrive

A

progressive loss of energy, strength, & stamina leading to decreased function/physical/cognitive deterioration

red flag: unplanned weight loss of 10% or more of body weight in last year

weakness, inability to care for self, dizziness, weight, memory loss & depression

184
Q

FTT labs

A
CBC
lytes
kidney & thyroid function 
fasting blood glucose 
LFTs
calcium 
UA
stool of OB x3
185
Q

elder abuse

A

physical, sexual, psychological, financial exploitation, neglect, abandonment, & self-neglect

1 in 6 adults > 60 y.o. have experienced abuse for > 1 yr

higher risk in those w/ disability/dementia

186
Q

elder abuse mgmt

A

home setting: report to state adult protective services

Long term setting: report to long term care ombudman/police if immediate risk

187
Q

parkinson dx RF

A

fam hx (1st degree relative)
pesticide exposure
concussion hx
urban/industrial areas w/ high copper, Mg, & lead levels
hydrocarbon solvent exposure (trichloroethylene)
milk consumption
excess body weight
anemia
older age - 70 y.o.
illict substances: mescaline, ecstacy, acid, those that cause dopamine surges causes greater risk of PD w/ hx of hallucinogenic abuse

188
Q

protective factor of parkinson dx

A

smoking cigarettes

stimulates nicotinic receptors lowers incidence of PD

189
Q

Parkinson dx presentation

A

cardinal features: resting asymmetric/unilateral tremor
bradykinesia (w/ freezing)
rigidity
postural changes/instability

r/t dopamine depletion

190
Q

motor symptoms of Parkinson

A
TRAP:
Tremor
Rigidity (classic increased resistance to passive movement of a joint in 90% pts)
Akinesia (slow movement like a sloth)
Postural instability (falling)

resting hand tremor “pill rolling” increases w/ walking (can be earliest sign)

masked facies (decreased blink rate/facial expression)

hypophonia (soft speech)

Aprosody of speech (monotone), dysarthria, palilalia (repetition of words/phrases)

191
Q

non-motor symptoms of Parkinson

A

excessive salvation, diff swallowing & speaking
forgetfulness, constipation, urinary retention
hyposmia (reduce/altered sense of smell

192
Q

parkinson comorbid complications

A

depression, psychosis

daytime sleepiness, fatigue

193
Q

parkinson dx diagnostics

A

gold standard: post-mortem neuropathic exam

rest tremor w/ unilateral onset & excellent response to dopaminergic therapy is important criteria

194
Q

parkinson pharm tx

A

Carbidopa/Levodopa (Sinemet) most effective drug for tremor, rigidity symptoms

Dopamine agonist (Requip, Mirapex)

MAO B (selegiline or rasagiline)

Anti viral drug (Amantadine [Symmetrel])

anticholinergics

195
Q

Parkinson dx surgical tx

A

deep brain stimulation

  • almost a cure, not quite a cure but controls people
  • can pull back med doses in pts
  • contraindication = space occupying lesion in brain, dementia (need to be cleared from psychiatry)
196
Q

Parkinson dx non-pharm mgmt

A

speech therapy, dance therapy
cognitive exercises
exercise & PT

197
Q

features that suggest alternate dx, not parkinson

A
falls early in dx
poor levodopa response 
symmetric motor signs 
lack of tremor 
early dysautonomia
198
Q

What are the 7 syndromes of geriatrics?

A

DRIVING, FRAGILITY, DEHYDRATION, FALLS, POLYPHARMACY, HEARING LOSS, ABUSE

199
Q

HOW OFTEN SHOULD YOU SCREEN FOR A PSA? WHAT AGE GROUP?

A

55-69 YO, Q 2-3 YEARS

200
Q

WHAT ARE RED FLAGS FOR POTENTIAL ABUSE IN THE ELDERLY POPULATION?

WHAT ARE FORMS OF ABUSE?

A

HOARDING, ANXIOUS, BRUISING IN VARIOUS STAGES, INAPPROPRIATE CLOTHING.

FINANCIAL, PHYSICAL, EMOTIONAL, NEGLECT.

201
Q

WHAT ARE SIGNS OF DEHYDRATION?

HOW WOULD YOU CALCULATE THEIR FLUID DEFICIT?

A

LOW SODIUM, HIGH H&H, ELECTROLYTE IMBALANCE, CREATININE/BUN ELEVATED, DRY TONGUE, DRY MUCOUS MEMBRANES, CONFUSED, LOW SKIN TURGOR.

FLUID DEFICIT = PRE -ILLNESS WT - CURRENT WEIGHT.
GIVE 1/2 NEEDED IN 24 HOURS, MIN 1.5L

202
Q

WHEN SHOULD A PATIENT BE SCREENED FOR A COLONOSCOPY? WHAT IF THEY HAVE IBD? WHAT IF THEIR MOM HAD COLON CANCER AT 45

A

45-75 YEAR OLD, Q 10 YEARS

IF THEY HAVE IBD Q8 YEARS after dx THEN EVERY 3-5 YEARS.

IF PATIENT HAS A 1ST DEGREE RELATIVE SCREEN 10 YEARS BEFORE THEIR FAMILY MEMBER’S DIAGNOSIS THEN REPEAT EVERY 5 YEARS.

203
Q

WHAT DIAGNOSTIC TEST SHOULD YOU OBTAIN IF THE PATIENT EXPERIENCES A TIA?

A

CAROTID ULTRASOUND

204
Q

AN 88 YEAR OLD PATIENT WITH A HISTORY OF HYPERTENSION AND DEPRESSION PRESENTS WITH VISUAL HALLUCINATIONS, CHANGES IN CONCENTRATION AND INCREASING LOSS OF COORDINATION THE PAST 2 MONTHS. WHAT IS YOUR MAIN DIFFERENTIAL?

A

LEWY BODY DEMENTIA

205
Q

WHAT ARE SIGNS AND SYMPTOMS OF A STROKE?

A

UNILATERAL WEAKNESS, NUMBNESS OF FACE, ARM, OR LEG, SLURRED SPEECH, BLURRY VISION, TROUBLE WALKING, DIZZINESS, LOSS OF BALANCE OR COORDINATION, SUDDEN SEVERE HEADACHE WITH NO KNOWN CAUSE

206
Q

A 68 YEAR OLD PATIENT WITH DIABETES, HYPERTENSION AND HYPERLIPIDEMIA IS IN THE HOSPITAL FOR AN ACUTE ISCHEMIC STROKE. HIS NIH SCORE IS 12. WHAT WOULD YOU ANTICIPATE HIS DISCHARGE PLAN WILL BE?

A

ACUTE INPATIENT REHAB

207
Q

A 78 YEAR OLD WOMAN PRESENTS TO THE ER FROM HER ASSISTED LIVING FACILITY WITH ACUTE FLUCTUATING COGNITIVE CHANGES. SHE HAS A PAST MEDICAL HISTORY OF DIABETES, FREQUENT YEAST INFECTIONS AND HYPOTHYROIDISM. WHAT DO YOU SUSPECT? WHAT IS THE MOST COMMON CAUSE OF THIS CONDITION?

A

DELIRIUM DUE TO INFECTION, POSSIBLY UTI

208
Q

WHAT IS CONSIDERED AN EARLY SIGN OF PARKINSON’S DISEASE?

A

RESTING, UNILATERAL TREMOR`

209
Q

WHAT IS CONSIDERED THE GOLD STANDARD FOR DIAGNOSING PD? HOW IS IT TYPICALLY DIAGNOSED?

A

POST MORTEM NEUROPATHIC EXAM,

CLINICALLY DIAGNOSED, RESTING UNILATERAL TREMOR, EXCELLENT RESPONSE TO DOPAMINERGIC THERAPY.

210
Q

WHICH MEDICATION REDUCES FREEZING in PD?

A

AMANTADINE

211
Q

NAME THE FOUR TYPES OF MEDICATIONS TO TREAT PARKINSON’S. WHICH IS THE GOLD STANDARD? IN SOMEONE WITH EARLY ONSET PD, WHAT WOULD YOU START WITH?

A

LEVADOPA (SINEMET) GOLD STANDARD, MOA-B (SELEGILINE), DOPAMINE AGONISTS (REQUIP), AMANTADINE. LEVADOPA ONLY WORKS FOR APPROXIMATELY 15 YEARS, START ON A DIFFERENT MEDICATION IN YOUNGER PATIENTS.

212
Q

WHAT EXAM FINDINGS COULD YOU SEE IN A PD PATIENT?

A

TREMOR, RIGIDITY, AKINESIA, POSTURAL INSTABILITY, MASKED FACIES, HYPOPHONIA, EXCESSIVE SALIVATION, DIFFICULTY SWALLOWING, CONSTIPATION, URINARY RETENTION, HYPOSMIA, DEPRESSION, DAYTIME SLEEPINESS, COGWHEELING RIGIDITY, PARKINSON SHUFFLE

213
Q

HTN screening older adult

A

each office visit, no age restriction

214
Q

DM screening older adult

A

BG in overweight/obese until age 70

215
Q

lipid screening older adult

A

fasting lipid panel every 5 years unless levels are high or CV RF

216
Q

height & wt older adults

A

each visit

217
Q

AAA screening

A

abdominal aorta US in males who have ever smoked after 65 y.o.

218
Q

cervical cancer screening older adults

A

none after age 65

219
Q

breast mammogram screening older adults

A

every 2 years until 74

220
Q

PSA screening older adults

A

PSA in M 55- 70 based on symptoms/preference

NOT recc > 70 y.o.

221
Q

bone mass screening

A

once at age 65 (F)

once age 70 (M)

222
Q

colorectal cancer screening

A

45- 75 y.o. recc

76-85 y.o selectively screen